In this paragraph we introduce and discuss the notion of an inertial coordinate system. The mathematical statements of this paragraph are formulated exactly in the next paragraph.
In this paragraph we define and investigate the galilean group of space-time transformations. Then we consider Newton’s equation and the simplest constraints imposed on its right-hand side by the property of invariance with respect to galilean transformations.3
We have already remarked that the form of the function F in Newton’s equation (1) is determined experimentally for each mechanical system. Here are several examples.In examining concrete systems it is reasonable not to include all the objects of the universe in a system. For example, in studying the majority of phenomena taking place on the earth we can ignore the influence of the moon. Furthermore, it is usually possible to disregard the effect of the processes we are studying on the motion of the earth itself; we may even consider a coordinate system attached to the earth as “fixed.” It is clear that the principle of relativity no longer imposes the constraints found in Section 2 for equations of motion written in such a coordinate system. For example, near the earth there is a distinguished direction, the vertical.
In this paragraph we study the phase flow of the differential equation (1). A look at the graph of the potential energy is enough for a qualitative analysis of such an equation. In addition, Equation (1) is integrated by quadratures.
Analyzing a general potential system with two degrees of freedom is beyond the capability of modern science. In this paragraph we look at the simplest examples.
In this section we study the connection between work and potential energy.
We will see later that the invariance of an equation of a mechanical problem with respect to some group of transformations always implies a conservation law. A central field is invariant with respect to the group of rotations. The corresponding first integral is called the angular momentum.
The law of conservation of angular momentum lets us reduce problems about motion in a central field to problems with one degree of freedom. Thanks to this, motion in a central field can be completely determined.
In this paragraph we define the angular momentum relative to an axis and we show that, for motion in an axially symmetric field, it is conserved.All the results obtained for motion in a plane can be easily carried over to motions in space.
In this paragraph we prove the laws of conservation of energy, momentum, and angular momentum for systems of material points in E3.
In some cases it is possible to obtain important information from the form of the equations of motion without solving them, by using the methods of similarity and dimension. The main idea in these methods is to choose a change of scale (of time, length, mass, etc.) under which the equations of motion preserve their form.
For what follows, we will need some facts from the calculus of variations. A more detailed exposition can be found in “A Course in the Calculus of Variations” by M. A. Lavrentiev and L. A. Lusternik, M. L., 1938, or G. E. Shilov, “Elementary Functional Analysis,” MIT Press, 1974.
Here we indicate the variational principle whose extremals are solutions of Newton’s equations of motion in a potential system.We compare Newton’s equations of dynamicswith the Euler-Lagrange equation(1)
The Legendre transformation is a very useful mathematical tool: it transforms functions on a vector space to functions on the dual space. Legendre transformations are related to projective duality and tangential coordinates in algebraic geometry and the construction of dual Banach spaces in analysis. They are often encountered in physics (for example, in the definition of thermodynamic quantities).
Problem. Show that the domain of g can be a point, a closed interval, or a ray if f is defined on the whole x axis. Prove that if f is defined on a closed interval, then g is defined on the whole p axis.
By means of a Legendre transformation, a lagrangian system of second-order differential equations is converted into a remarkably symmetrical system of 2n first-order equations called a hamiltonian system of equations (or canonical equations).
The phase flow of Hamilton’s equations preserves phase volume. It follows, for example, that a hamiltonian system cannot be asymptotically stable.
In this paragraph we define the notion of a system of point masses with holonomic constraints.
The configuration space of a system with constraints is a differentiable manifold. In this paragraph we give the elementary facts about differentiable manifolds.
Example 1. Euclidean space ℝn is a manifold, with an atlas consisting of one chart.Example 2. The sphere S2 = {(x, y, z): x2 + y2 + z2 = 1} has the structure of a manifold, with atlas, for example, consisting of two charts (Ui, φi, i = 1, 2) in stereographic projection (Figure 57). An analogous construction applies to the n-sphere
Figure 57Atlas of a sphereExample 3. Consider a planar pendulum. Its configuration space—the circle S1—is a manifold. The usual atlas is furnished by the angular coordinates φ: ℝ1 → S1, U1 = (− π, π), U2 = (0, 2π) (Figure 58).Example 4. The configuration space of the “spherical” mathematical pendulum is the two-dimensional sphere S2 (Figure 58).Example 5. The configuration space of a“planar double pendulum” is the direct product of two circles, i.e., the two-torus T2 = S1 × S1 (Figure 58).Example 6. The configuration space of a spherical double pendulum is the direct product of two spheres, S2 × S2.Example 7. A rigid line segment in the (q1, q2)-plane has for its configuration space the manifold ℝ2 × S1, with coordinates q1, q2, q3 (Figure 59). It is covered by two charts.
Figure 59Configuration space of a segment in the planeExample 8. A rigid right triangle O AB moves around the vertex O. The position of the triangle is given by three numbers: the direction OA ∈ S2 is given by two numbers, and if OA is given, one can rotate OB ∈ S1 around the axis OA (Figure 60).
Figure 60Configuration space of a triangleConnected with the position of the triangle O AB is an orthogonal right-handed frame, e1 = OA/|OA|, e2 = OB/|OB|, e3 = [e1, e2]. The correspondence is one-to-one; therefore the position of the triangle is given by an orthogonal three-by-three matrix with determinant 1.The set of all three-by-three matrices is the nine-dimensional space ℝ9. Six orthogonality conditions select out two three-dimensional connected manifolds of matrices with determinant + 1 and −1. The rotations of three-space (determinant + 1) form a group, which we call SO(3).Therefore, the configuration space of the triangle OAB is SO(3).Problem. Show that SO(3) is homeomorphic to three-dimensional real projective space.
Example 9. Consider a system of k rods in a closed chain with hinged joints.Problem. How many degrees of freedom does this system have?Example 10. Embedded manifolds. We say that M is an embedded k-dimensional sub-manifold of euclidean space ℝn (Figure 61) if in a neighborhood U of every point x ∈ M there are n − k functions f1: U → ℝ, f2: U → ℝ,..., fn − k: U → ℝ such that the intersection of U with M is given by the equations f1 = 0,..., fn − k = 0, and the vectors grad f1,..., grad fn − k at x are linearly independent.
Figure 61Embedded submanifoldIt is easy to give M the structure of a manifold, i.e., coordinates in a neighborhood of x(how?).It can be shown that every manifold can be embedded in some euclidean space. In Example 8, SO(3) is a subset of ℝ9.Problem. Show that SO(3) is embedded in ℝ9, and at the same time, that SO(3) is a manifold.
It is easy to define the operations of multiplication of a tangent vector by a number and addition of tangent vectors. The set of tangent vectors to M at x forms a vector space TMx. This space is also called the tangent space to M at x.
For example, the length of a curve on a manifold is expressed using this form as l(γ) =, or, if the curve is given parametrically, γ: [t0, t1] → M, t → x(t) ∈ M, then
Problem. Show that the vector f*xv does not depend on the curve φ, but only on the vector v.Problem. Show that the map f*x: TMx → TNf(x) is linear.Problem. Let x = (x1,..., xm) be coordinates in a neighborhood of x ∈ M, and y = (y1,..., yn) be coordinates in a neighborhood of y ∈ N. Let ξ be the set of components of the vector v, and η the set of components of the vector f*xv. Show thatTaking the union of the mappings f*x for all x, we get a mapping of the whole tangent bundleProblem. Show that f* is a differentiable map.Problem. Let f: M → N, g: N → K, and h = g ◦ f: M → K. Show that h* = g* ◦ f*.
In this paragraph we define lagrangian dynamical systems on manifolds. Systems with holonomic constraints are a particular case.
Example. Let M be a region in a coordinate space with coordinates q = (q1,...,qn). The lagrangian function L: TM → ℝ may be written in the form of a functionof the 2n coordinates. As we showed in Section 12, the evolution of coordinates of a point moving with time satisfies Lagrange’s equations.
Example. Consider two mass points m1 and m2 joined by a line segment of length l in the (x, y)-plane. Then a configuration space of three dimensionsis defined in the four-dimensional configuration space ℝ2 × ℝ2 of two free points (x1, y1) and (x2, y2) by the condition(Figure 65).
Figure 65Segment in the planeThere is a quadratic form on the tangent space to the four-dimensional space (x1, x2, y1, y2):Our three-dimensional manifold, as it is embedded in the four-dimensional one, is provided with a Riemannian metric. The holonomic system thus obtained is called in mechanics a line segment of fixed length in the (x, y)-plane. The kinetic energy is given by the formula
Example. We consider the motion of a point mass of mass 1 on a surface of revolution in three-dimensional space. It can be shown that the orbits are geodesics on the surface. In cylindrical coordinates r, φ, z the surface is given (locally) in the form r = r(z) or z = z(r). The kinetic energy has the form (Figure 66)in coordinates φ and z, andin coordinates r and φ. (We have used the identity.)
Figure 66Surface of revolutionThe lagrangian function L is equal to T. In both coordinate systems φ is a cyclic coordinate. The corresponding momentum is preserved;is nothing other than the z-component of angular momentum. Since the system has two degrees of freedom, knowing the cyclic coordinate φ is sufficient for integrating the problem completely (cf. Corollary 3, Section 15).
We can obtain more easily a clear picture of the orbits by reasoning slightly differently. Denote by α the angle of the orbit with a meridian. We have, where |v| is the magnitude of the velocity vector (Figure 66).
By the law of conservation of energy, H = L = T is preserved. Therefore, |v| = const, so the conservation law for pφ takes the form(“Clairaut’s theorem”).This relationship shows that the motion takes place in the region |sin α| ≤ 1, i.e., r ≥ r0 sin α0. Furthermore, the inclination of the orbit from the meridian increases as the radius r decreases. When the radius reaches the smallest possible value, r = r0 sin α0, the orbit is reflected and returns to the region with larger r (Figure 67).
Figure 67Geodesics on a surface of revolutionProblem. Show that the geodesics on a convex surface of revolution are divided into three classes: meridians, closed curves, and geodesics dense in a ring r ≥ c.Problem. Study the behavior of geodesics on the surface of a torus ((r − R)2 + z2 = ρ2).
Example. Consider the motion of a bead along a vertical circle of radius r (Figure 68) which rotates with angular velocity ω around the vertical axis passing through the center O of the circle. The manifold M is the circle. Let q be the angular coordinate on the circle, measured from the highest point.Let x, y, and z be cartesian coordinates in E3 with origin O and vertical axis z. Let φ be the angle of the plane of the circle with the plane xOz. By hypothesis, φ = ωt. The mapping i: M × ℝ → E3 is given by the formulaFrom this formula (or, more simply, from an “infinitesimal right triangle”) we find thatIn this case the lagrangian function L = T − U turns out to be independent of t, although the constraint does depend on time. Furthermore, the lagrangian function turns out to be the same as in the one-dimensional system with kinetic energyand with potential energyThe form of the phase portrait depends on the ratio between A and B. For 2B < A (i.e., for a rotation of the circle slow enough that ω2r < g), the lowest position of the bead (q = π) is stable and the characteristics of the motion are generally the same as in the case of a mathematical pendulum (ω = 0).For 2B > A, i.e., for sufficiently fast rotation of the circle, the lowest position of the bead becomes unstable; on the other hand, two stable positions of the bead appear on the circle, where cos q = −A/2B = −g/ω2r. The behavior of the bead under all possible initial conditions is clear from the shape of the phase curves in the(Figure 69).
Figure 69Effective potential energy and phase plane of the bead
Various laws of conservation (of momentum, angular momentum, etc.) are particular cases of one general theorem: to every one-parameter group of diffeomorphisms of the configuration manifold of a lagrangian system which preserves the lagrangian function, there corresponds a first integral of the equations of motion.
Example. Let. The system admits the translation h: (x1, x2, x2) → (x1 + s, x1, x3) along the x1 axis and does not admit, generally speaking, translations along the x2 axis.
Problem 1. Suppose that a particle moves in the field of the uniform helical line x = cos φ, y = sin φ, z = cφ. Find the law of conservation corresponding to this helical symmetry.Answer. In any system which admits helical motions leaving our helical line fixed, the quantity I = cP3 + M3 is conserved.Problem 2. Suppose that a rigid body is moving under its own inertia. Show that its center of mass moves linearly and uniformly. If the center of mass is at rest, then the angular momentum with respect to it is conserved.Problem 3. What quantity is conserved under the motion of a heavy rigid body if it is fixed at some point O? What if, in addition, the body is symmetric with respect to an axis passing through O?Problem 4. Extend Noether’s theorem to non-autonomous lagrangian systems.Hint. Let M1 = M × ℝ be the extended configuration space (the direct product of the configuration manifold M with the time axis ℝ).Define a function L1: TM1 byi.e., in local coordinates q, t on M1 we define it by the formulaWe apply Noether’s theorem to the lagrangian system (M1, L1).If L1 admits the transformations hs: M1 → M1, we obtain a first integral I1: TM1 → ℝ. Since ∫ L dt = ∫L1 dτ. this reduces to a first integral I: TM × ℝ → ℝ of the original system. If, in local coordinates (q, t) on M1 we have I1 = I1(q, t, dq/dτ, dt/dτ), then.
In particular, if L does not depend on time, L1 admits translations along time, hs(q, t) = (q, t + s). The corresponding first integral I is the energy integral.
We give here a new definition of a system of point masses with holonomic constraints and prove its equivalence to the definition given in Section 17.
The physical meaning of the constraint force becomes clear if we consider our system with constraints as the limit of systems with potential energy U + NU1 as N → ∞, where U1(x) = ρ2(x, M). For large N the constraint potential NU1 produces a rapidly changing force F = −N ∂U1/∂x; when we pass to the limit (N → ∞) the average value of the force F under oscillations of x near M is R. The force F is perpendicular to M. Therefore, the constraint force R is perpendicular to M: (R, ξ) = 0 for every tangent vector ξ.
If we define a system with holonomic constraints as a limit as N → ∞ then the D’Alembert-Lagrange principle becomes a theorem: its proof is sketched above for the simplest case.It is possible, however, to define an ideal holonomic constraint using the D’Alembert-Lagrange principle. In this way we have three definitions of holonomic systems with constraints:1.The limit of systems with potential energies U + NU1 as N → ∞.2.A holonomic system (M, L), where M is a smooth submanifold of the configuration space of a system without constraints and L is the lagrangian.3.A system which complies with the D’Alembert-Lagrange principle.All three definitions are mathematically equivalent.The proof of the implications (1) ⟹ (2) and (1) ⟹ (3) is sketched above and will not be given in further detail. We will now show that (2) ⟺ (3).
Problem. A rod of weight P, tilted at an angle of 60° to the plane of a table, begins to fall with initial velocity zero (Figure 74). Find the constraint force of the table at the initial moment, considering the table as (a) absolutely smooth and (b) absolutely rough. (In the first case, the holonomic constraint holds the end of the rod on the plane of the table, and in the second case, at a given point.)
Figure 74Constraint force on a rod
Problem. Show that in an analytic system with one degree of freedom an equilibrium position q0 which is not a strict local minimum of the potential energy is not stable in the sense of Liapunov. Produce an example of an infinitely differentiable system where this is not true.
Problem. Find the period of small oscillations of a bead of mass 1 on a wire y = U(x) in a gravitational field with g = 1, near an equilibrium position x = x0 (Figure 77).Solution. We have
Figure 77Bead on a wireLet x0 be a stable equilibrium position: (∂U/∂x)|x0 = 0; (∂2U/∂x2)|x0 > 0. Then the frequency of small oscillations, ω, is defined by the formulasince, for the linearized system,.
Problem. Show that not only a small oscillation, but any motion of the bead is equivalent to a motion in some one-dimensional system with lagrangian function.
Hint. Take length along the wire for q.
Example 1. Consider the system of two identical mathematical pendulums of length l1 = l2 = 1 and mass m1 = m2 = 1 in a gravitational field with g = 1. Suppose that the pendulums are connected by a weightless spring whose length is equal to the distance between the points of suspension (Figure 79). Denote by q1 and q2 the angles of inclination of the pendulums. Thenfor small oscillations,
Figure 79Identical connected pendulumsand
, where
is the potential energy of the elasticity of the spring. Set
Thenand both forms are reduced to principal axes:where ω1 = 1 and(Figure 80). So the two characteristic oscillations are as follows (Figure 81):
1.Q2 = 0, i.e., q1 = q2; both pendulums move in phase with the original frequency 1, and the spring has no effect;2.Q1 = 0, i.e., q1 = −q2: the pendulums move in opposite phase with increased frequency ω2 > 1 due to the action of the spring.
Figure 80Configuration space of the connected pendulums
Figure 81Characteristic oscillations of the connected pendulumsExample 2. Suppose that the pendulums are at rest at the initial moment, and one of them is given velocity. We will show that after some time T the first pendulum will be almost stationary, and all the energy will have gone to the second.
It follows from the initial conditions that Q1(0) = Q2(0) = 0. Therefore, Q1 = c1 sin t, and Q2 = c2 sin ωt with. But
. Therefore,
and
, and our solution has the form
or, disregarding the term v(1 − (1/ω))sin ωt, which is small since α is,The quantity ε ≈ α/2 is small, since α is; therefore q1 undergoes an oscillation of frequency ω′ ≈ 1 with slowly changing amplitude v cos εt (Figure 82).
Figure 82Beats: trajectories in the configuration spaceAfter time T = π/2ε ≈ π/α, essentially only the second pendulum will be oscillating; after 2T, again only the first, etc. (“beats”) (Figure 83).
Figure 83BeatsExample 3. We investigate the characteristic oscillations of two different pendulums (m1 ≠ m2, l1 ≠ l2, g = 1), connected by a spring with energy(Figure 84). How do the characteristic frequencies behave as α → 0 or as α → ∞?
Figure 84Connected pendulumsWe haveTherefore (Figure 85),and the characteristic equation has the form
Figure 85Potential energy of strongly connected pendulumsorwhereThis is the equation of a hyperbola in the (α, λ)-plane (Figure 86). As α → 0 (weak spring) the frequencies approach the frequencies of free pendulums; as α → ∞, one of the
frequencies tends to ∞, while the other approaches the characteristic frequency ω∞ of a pendulum with two masses on one rod (Figure 87):
Figure 86Dependence of characteristic frequencies on the stiffness of the spring
Figure 87Limiting case of pendulums connected by an infinitely stiff springProblem. Investigate the characteristic oscillations of a planar double pendulum (Figure 88).
Figure 88Double pendulumProblem. Find the shape of the trajectories of the small oscillations of a point mass on the plane, sitting inside an equilateral triangle and connected by identical springs to the vertices (Figure 89).
Figure 89System with an infinite set of characteristic oscillationsSolution. Under rotation by 120° the system is mapped onto itself. Consequently, all directions are characteristic, and both characteristic frequencies are the same:. Therefore, the trajectories are ellipses (cf. Figure 20).
We prove here the Rayleigh-Courant-Fisher theorem on the behavior of characteristic frequencies of a system under increases in rigidity and under imposed constraints.
Problem. Discuss the one-dimensional case.
Example. Under an increase in the rigidity α of the spring connecting the pendulums of Example 3, Section 23, the potential energy grows, and by Theorem 1, the characteristic frequencies grow: dωi/dα > 0.Now consider the case when the rigidity of the spring approaches infinity, α → ∞. Then in the limit the pendulums are rigidly connected and we get a system with one degree of freedom; the limiting characteristic frequency ω∞ satisfies ω1 < ω∞ < ω2.
Problem. Show that under the orthogonal projection of an ellipsoid lying in one subspace of euclidean space onto another subspace, all the semi-axes are decreased.Problem. Suppose that a quadratic form A(ε) on euclidean space ℝn is a continuously differentiable function of the parameter ε. Show that every characteristic frequency depends differentiably on ε, and find the derivatives.Answer. Let λ1,...,λk be the eigenvalues of A(0). To every eigenvalue λi of multiplicity vi there corresponds a subspace ℝvi. The derivatives of the eigenvalues of A(ε) at 0 are equal to the eigenvalues of the restricted form B = (dA/dε)|ε = 0 on ℝvi.In particular, if all the eigenvalues of A(0) are simple, then their derivatives are equal to the diagonal elements of the matrix B in the characteristic basis for A(0).It follows from this problem that when a form is increased, its eigenvalues grow. In this way we obtain new proofs of Theorems 1 and 2.Problem. How does the pitch of a bell change when a crack appears in the bell?
Example. For the systemswhich can be considered periodic with any period T, the mapping A is a rotation or a hyperbolic rotation (Figure 96).
Figure 96Rotation and hyperbolic rotation
Problem. Find the shape of the region of stability in the ε,ω-plane for the system described by the equationsSolution. It follows from the solution of the preceding problem that A = A2 A1, whereTherefore, the boundary of the zone of stability has the equation(5)
Since, we have ω1/ω2 = (ω + ε)/(ω − ε) ≈ 1. We introduce the notation
Then, as is easily computed,. Using the relations 2c1c2 = cos 2πε + cos 2πω and 2s1s2 = cos 2πε − cos 2πω, we rewrite Equation (5) in the form
or(6a)
In the first case cos 2πω ≈ 1. Therefore, we set(6b)
We rewrite Equation (6a) in the formor 2π2a2 + O(a4) = Δπ2ε2 + O(ε4).Substituting in the value Δ = (2ε2/ω2) + O(ε4), we findEquation (6b) is solved analogously; for the result we getTherefore the answer has the form depicted in Figure 101.
Figure 101Zones of parametric resonance for f = ω ± ε.
Solution. The equation of motion can be written in the form ẋ = (ω2 ± d2)x (the sign changes after time τ), where ω2 = g/l and d2 = c/l. If the oscillation of the suspension is fast enough, then d2 > ω2 (d2 = 8a/lτ2).As in the previous problem, A = A2 A1, whereThe stability condition |tr A| < 2 therefore has the form(7)
We will show that this condition is fulfilled for sufficiently fast oscillations of the point of suspension, i.e., when. We introduce the dimensionless variables ε, μ:
ThenTherefore, for small ε and μ we have the following expansion with error o(ε4 + μ4):so the stability condition (7) takes the formi.e., disregarding the small higher-order terms,. This condition can be rewritten as
where N = 1/2τ is the number of oscillations of the point in one unit of time. For example, if the length of the pendulum l is 20 cm, and the amplitude of the oscillation of the point of suspension a is 1 cm, thenFor example, the topmost position is stable if the frequency of oscillation of the point of suspension is greater than 40 per second.
In this paragraph we define angular velocity.
Example. The angular velocity of the earth is directed from the center to the North Pole; its length is equal to 2π/3600 · 24sec−1 ≈ 7.3 · 10−5 sec−1.
Problem. A watch lies on a table. Find the angular velocity of the hands of the watch: (a) relative to the earth, (b) relative to an inertial coordinate system.
The equations of motion in a non-inertial coordinate system differ from the equations of motion in an inertial system by additional terms called inertial forces. This allows us to detect experimentally the non-inertial nature of a system (for example, the rotation of the earth around its axis).
Example 1. At the moment of takeoff, a rocket has accelerationdirected upward (Figure 107). Thus, the coordinate system K connected to the rocket is not inertial, and an observer inside can detect the existence of a force field mW and measure the inertial force, for example, by means of weighted springs. In this case the inertial force is called overload.*
Figure 107OverloadEXAMPLE 2. When jumping from a loft, a person has acceleration g, directed downwards. Thus, the sum of the inertial force and the force of gravity is equal to zero; weighted springs show that the weight of any object is equal to zero, so such a state is called weightlessness. In exactly the same way, weightlessness is observed in the free ballistic flight of a satellite since the force of inertia is opposite to the gravitational force of the earth.Example 3. If the point of suspension of a pendulum moves with acceleration W(t), then the pendulum moves as if the force of gravity g were variable and equal to g − W(t).
We will consider in more detail the effect of the earth’s rotation on laboratory experiments. Since the earth rotates practically uniformly, we can take. The centrifugal force has its largest value at the equator, where it attains Ω2ρ/g ≈ (7.3 × 10−5)2 · 6.4 × 106/9.8 ≈ 3/1000 the weight. Within the limits of a laboratory it changes little, so to observe it one must travel some distance. Thus, within the limits of a laboratory the rotation of the earth appears only in the form of the Coriolis force: in the coordinate system Q associated to the earth, we have, with good accuracy,
(the centrifugal force is taken into account in g).Example 1. A stone is thrown (without initial velocity) into a 250 m deep mine shaft at the latitude of Leningrad. How far does it deviate from the vertical?We solve the equationby the following approach, taking Ω ≪ 1. We set (Figure 109)whereand Q1 = Q1(0) + gt2/2. For Q2, we then get
Figure 109Displacement of a falling stone by Coriolis forceProblem. By how much would the Coriolis force displace a missile fired vertically upwards at Leningrad from falling back onto its launching pad, if the missile rose 1 kilometer?Example 2 (The Foucault pendulum). Consider small oscillations of an ideal pendulum, taking into account the Coriolis force. Let ex ey, and ez be the axes of a coordinate system associated to the earth, with ez directed upwards, and ex and ey in the horizontal plane (Figure 110). In the approximation of small oscillations,(in comparison with ẋ and ẏ); therefore, the horizontal component of the Coriolis force will be 2mẏΩzex − 2mẋΩzey. From this we get the equations of motion
Figure 110Coordinate system for studying the motion of a Foucault pendulumIf we set x + iy = w, then ẇ = ẋ + iẏ,, and the two equations reduce to one complex equation
We solve it: w = eλt, λ2 + 2iΩzλ + ω2 = 0,. But
. Therefore,
, from which it follows, by disregarding
, that
or, to the same accuracy,For Ωz = 0 we get the usual harmonic oscillations of a spherical pendulum. We see that the effect of the Coriolis force reduces to a rotation of the whole picture with angular velocity −Ωz, where |Ω| = |Ω| sin λ0.In particular, if the initial conditions correspond to a planar motion (y(0) = ẏ(0) = 0), then the plane of oscillation will be rotating with angular velocity −Ωz with respect to the earth’s coordinate system (Figure 111).
Figure 111Trajectory of a Foucault pendulumAt a pole, the plane of oscillation makes one turn in a twenty-four-hour day (and is fixed with respect to a coordinate system not rotating with the earth). At the latitude of Moscow (56°) the plane of oscillation turns 0.83 of a rotation in a twenty-four-hour day, i.e., 12.5° in an hour.Problem. A river flows with velocity 3 km/hr. For what radius of curvature of a river bend is the Coriolis force from the earth’s rotation greater than the centrifugal force determined by the flow of the river?Answer. The radius of curvature must be least on the order of 10 km for a river of medium width.The solution of this problem explains why a large river in the northern hemisphere (for example, the Volga in the middle of its course), undermines the base of its right bank, while a river like the Moscow River, with its abrupt bends of small radius, undermines either the left or right (whichever is outward from the bend) bank.
In this paragraph we define a rigid body and its inertia tensor, inertia ellipsoid, moments of inertia, and axes of inertia.
Example. The inertia ellipsoid of three points of mass m at the vertices of an equilateral triangle with center 0 is an ellipsoid of revolution around an axis normal to the plane of the triangle (Figure 117).
Figure 117Ellipsoid of inertia of an equilateral triangle
Problem. Draw the line through the center of a cube such that the sum of the squares of its distances from the vertices of the cube is: (a) largest, (b) smallest.
Example. Find the principal axes and moments of inertia of the uniform planar plate |x| ≤ a, |y| < b, z = 0 with respect to O.Solution. Since the plate has three planes of symmetry, the inertia ellipsoid has the same planes of symmetry and, therefore, principal axes x, y, and z. Furthermore,In the same wayClearly, Iz = Ix + Iy.Problem. Show that the moments of inertia of any body satisfy the triangle inequalitiesand that equality holds only for a planar body.Problem. Find the axes and moments of inertia of a homogeneous ellipsoid of mass m with semiaxes a, b, and c relative to the center O.Hint. First look at the sphere.
Problem. Find the principal axes and moments of inertia of a uniform tetrahedron relative to its vertices.Problem. Draw the angular momentum vector M for a body with a given inertia ellipsoid rotating with a given angular velocity Ω.Answer. M is in the direction normal to the inertia ellipsoid at a point on the Ω axis (Figure 119).
Figure 119Angular velocity, ellipsoid of inertia and angular momentumProblem. A piece is cut off a rigid body fixed at the stationary point O. How are the principal moments of inertia changed? (Figure 120).Answer. All three principal moments are decreased.Hint. Cf. Section 24.Problem. A small mass ε is added to a rigid body with moments of inertia I1 > I2 > I3 at the point Q = x1e1 + x2e2 + x3e3. Find the change in I1 and e1 with error O(ε2).Solution. The center of mass is displaced by a distance of order ε. Therefore, the moments of inertia of the old body with respect to the parallel axes passing through the old and new centers of mass differ in magnitude of order ε2. At the same time, the addition of mass changes the moment of inertia relative to any fixed axis by order ε. Therefore, we can disregard the displacement of the center of mass for calculations with error O(ε2).Thus, after addition of a small mass the kinetic energy takes the formwhereis the kinetic energy of the original body. We look for the eigenvalue I1(ε) and eigenvector e1(ε) of the inertia operator in the form of a Taylor series in ε. By equating coefficients of ε in the relation A(ε)e1(ε) = I1(ε)e1(ε), we find that, within error O(ε2):
From the formula for I1(ε) it is clear that the change in the principal moments of inertia (to the first approximation in ε) is as if neither the center of mass nor the principal axes changed. The formula for e1(ε) demonstrates how the directions of the principal axes change: the largest principal axis of the inertia ellipsoid approaches the added point, and the smallest recedes from it. Furthermore, the addition of a small mass on one of the principal planes of the inertia ellipsoid rotates the two axes lying in this plane and does not change the direction of the third axis. The appearance of the differences of moments of inertia in the denominator is connected with the fact that the major axes of an ellipsoid of revolution are not defined. If the inertia ellipsoid is nearly an ellipsoid of revolution (i .e., I1 ≈ I2) then the addition of a small mass could strongly turn the axes e1 and e2 in the plane spanned by them.
Here we study the motion of a rigid body around a stationary point in the absence of outside forces and the similar motion of a free rigid body. The motion turns out to have two frequencies.
Problem. Are stationary rotations of the body around the largest and smallest principal axes Liapunov stable?Answer. No.
Problem. Find the angular velocity of precession.Answer. Decompose the angular velocity vector ω into components in the directions of the angular momentum vector m and the axis of the body Bte1. The first component gives the angular velocity of precession, ωpr = M/I2.
Hint. Represent the motion of the body as the product of a rotation around the axis of momentum and a subsequent rotation around the axis of the body. The sum of the angular velocity vectors of these rotations is equal to the angular velocity vector of the product.Remark. In the absence of outside forces, a rigid body fixed at a point O is represented by a lagrangian system whose configuration space is a group, namely SO(3), and the lagrangian function is invariant under left translations. One can show that a significant part of Euler’s theory of rigid body motion uses only this property and therefore holds for an arbitrary left-invariant lagrangian system on an arbitrary Lie group. In particular, by applying this theory to the group of volume-preserving diffeomorphisms of a domain D in a riemannian manifold, one can obtain the basic theorems of the hydrodynamics of an ideal fluid. (See Appendix 2.)
We consider here the motion of an axially symmetric rigid body fixed at a stationary point in a uniform force field. This motion is composed of three periodic processes: rotation, precession, and nutation.
ex, ey, and ez
|
are the unit vectors of a right-handed cartesian stationary coordinate system at the stationary point O;
|
e1, e2, and e3
|
are the unit vectors of a right moving coordinate system connected to the body, directed along the principal axes at O;
|
I1 = I2 ≠ I3 eN
|
are the moments of inertia of the body at O; is the unit vector of the axis [ez, e3], called the “line of nodes” (all vectors are in the “stationary space” k).
|
The formulas obtained in Section 30 reduce the solution of the equations of motion of a top to elliptic integrals. However, qualitative information about the motion is usually easy to obtain without turning to quadrature.In this paragraph we investigate the stability of a vertical top and give approximate formulas for the motion of a rapidly spinning top.
Problem. Show that a stationary rotation around the vertical axis is always Liapunov unstable.
Problem. Show that, for, the axis of a sleeping top is stable with respect to perturbations which change the values of Mz and M3, as well as θ.
Proof. In the absence of gravity the effective potential energy reduces toThis nonnegative function has the minimum value of zero for the angle θ = θ0 determined by the condition Mz = M3 cos θ0 (Figure 130). Thus, the angle of inclination θ0 of the top’s axis to the vertical is stably stationary: for small deviations of the initial angle θ from θ0, there will be periodic oscillations of θ near θ0 (nutation). The frequency of these oscillations is easily determined by the following general formula: the frequency ω of small oscillations in a one-dimensional system with energyis given (Section 22D) by the formulaThe energy of the one-dimensional system describing oscillations of the inclination of the top’s axis isFor θ = θ0 + x we find Mz −M3cosθ =M3(cosθ0 −cos(θ0 + x)) = M3x sin θ0 + O(x2)from which we obtain the expression for the frequency of nutation□
Figure 130Effective potential energy of a top
Remark. Now the motion of the top’s axis, which according to Lagrange was called nutation, is called precession in Poinsot’s description of motion.
Lemma. Suppose that the function f(x) has a minimum at x = 0 and Taylor expansion f(x) = Ax2/2 + ..., A > 0. Suppose that the function h(x) has Taylor expansion h(x) = B + Cx + ⋯. Then, for sufficiently small ε, the function fε(x) = f(x) + εh(x) has a minimum at the point (Figure 132)which is close to zero. In addition,.
Proof. We haveand the result is obtained by applying the implicit function theorem to
. ☐
Figure 132Displacement of the minimum under a small change of the function
We already know from Section 30C that under our initial conditions the axis of the top traces a curve with cusps on the sphere.We apply the lemma to locate the minimum point θg of the effective potential energy. We set (Figure 133)Then we obtain, as above, the Taylor expansion in x at θ0Applying the lemma to f = Ueff|g = 0, g = ε, h = ml cos(θ0 + x), we find that the minimum of the effective potential energy Ueff is attained at angle of inclinationThus the inclination θ of the top’s axis will oscillate near θg (Figure 134). But, at the initial moment, θ = θ0 and. This means that θ0 corresponds to the highest position of the axis of the top. Thus, for small g, the amplitude of nutation is asymptotically equal to
We now find the precessional motion of the axis. From the general formulafor Mz = M3cosθ0 and θ = θ0 + x, we find that Mz − M3cosθ = M3 xsinθ0 + ⋯; soBut x oscillates harmonically between 0 and 2xg (up to O(g2)). Therefore, the average value of the velocity of precession over the period of nutation is asymptotically equal to
Figure 133Definition of the amplitude of nutation
Figure 134Motion of a top’s axis
Here we define exterior algebraic forms
Example. If a uniform force field F is given on euclidean ℝ3, its work A on the displacement ξ is a 1-form acting on ξ (Figure 135).
Figure 135The work of a force is a 1-form acting on the displacement.
Example 1. Let S(ξ1, ξ2) be the oriented area of the parallelogram constructed on the vectors ξ1 and ξ2 of the oriented euclidean plane ℝ2, i.e.,with e1, e2 a basis giving the orientation on ℝ2.Example 2. Let v be a uniform velocity vector field for a fluid in three-dimensional oriented euclidean space (Figure 137). Then the flux of the fluid over the area of the parallelogram ξ1, ξ2 is a bilinear skew symmetric function of ξ1 and ξ2, i.e., a 2-form defined by the triple scalar product
Figure 137Flux of a fluid through a surface is a 2-form.Example 3. The oriented area of the projection of the parallelogram with sides ξ1 and ξ2 on the x1, x2-plane in euclidean ℝ3 is a 2-form.Problem 1. Show that for every 2-form ω2 on ℝn we haveSolution. By skew symmetry, ω2(ξ, ξ) = −ω2(ξ, ξ).
Problem 2. Show that this space is finite-dimensional, and find its dimension.Answer. n(n − 1)/2; a basis is shown below.
Example 1. The oriented volume of the parallelepiped with edges ξ1,..., ξn in oriented euclidean space ℝn is an n-form (Figure 138).where ξi = ξi1e1 + ⋯ + ξinen and e1,..., en are a basis of ℝn.Example 2. Let ℝk be an oriented k-plane in n-dimensional euclidean space ℝn. Then the k-dimensional oriented volume of the projection of the parallelepiped with edges ξ1, ξ2,..., ξk ∈ ℝn onto ℝk is a k-form on ℝn.
Problem 3. Show that this vector space is finite-dimensional and find its dimension.Answer.: a basis is shown below.
Problem 4. Show that ω1 ⋀ ω2 really is a 2-form.Problem 5. Show that the mappingis bilinear and skew symmetric:Hint. The determinant is bilinear and skew-symmetric not only with respect to rows, but also with respect to columns.
Problem 6. Show that theforms xi ⋀ xj (i < j) are linearly independent.
Problem 7. Show that every 2-form in the three-dimensional space (x1, x2, x3) is of the formProblem 8. Show that every 2-form on the n-dimensional space with coordinates x1,..., xn can be uniquely represented in the formHint. Let ei be the i-th basis vector, i.e., xi(ei) = 1, xj(ei) = 0 for i ≠ j. Look at the value of the form ω2 on the pair ei, ej. Then
Problem 9. Show that ω1 ⋀ ⋯ ⋀ ωk is a k-form.Problem 10. Show that the operation of exterior product of 1-forms gives a multi-linear skew-symmetric mappingIn other words,andwhere
Problem 11. Show that, if two of the indices i1,..., ik are the same, then the form xi1⋀ ⋯ ⋀ xik is zero.Problem 12. Show that the formsare linearly independent.The number of such forms is clearly. We will call them basic k-forms.
Problem 13. Show that every k-form on ℝn can be uniquely represented as a linear combination of basic forms:Hint. a i 1,..., ik = ωk(ei1, .., eik).
Problem 14. Show that every k-form on ℝn with k > n is zero.
Problem 15. Show that the product of monomials is associative:and skew-commutative:Hint. In order to move each of the l factors of ωl forward, we need k inversions with the k factors of ωk.Remark. It is useful to remember that skew-commutativity means commutativity only if one of the degrees k and l is even, and anti-commutativity if both degrees k and l are odd.
We define here the operation of exterior multiplication of forms and show that it is skew-commutative, distributive, and associative.
Example. If k = l = 1, then there are just two partitions: ξ1, ξ2 and ξ2, ξ1. Therefore,which agrees with the definition of multiplication of 1-forms in Section 32.Problem 1. Show that the definition above actually defines a k + l-form (i.e., that the value of (ωk ⋀ ωl)(ξ1,...,ξk+l) depends linearly and skew-symmetrically on the vectors ξ).
Problem 2. Show that the exterior square of a 1-form, or, in general, of a form of odd order, is equal to zero: ωk ⋀ ωk = 0 if k is odd.Example 1. Consider a coordinate system p1,..., pn, q1,..., qn on ℝ2n and the 2-form.[Geometrically, this form signifies the sum of the oriented areas of the projection of a parallelogram on the n two-dimensional coordinate planes (p1, q1),.... (pn, qn). Later, we will see that the 2-form ω2 has a special meaning for hamiltonian mechanics. It can be shown that every nondegenerate54 2-form on ℝ2n has the form ω2 in some coordinate system (p1,..., qn).]Problem 3. Find the exterior square of the 2-form ω2.Answer.
Problem 4. Find the exterior k-th power of ω2.Answer.Example 2. Consider the oriented euclidean space ℝ3. Every vector A ∈ ℝ3 determines a 1-form(scalar product) and a 2-form
by
Problem 5. Show that the mapsand
establish isomorphisms of the linear space ℝ3 of vectors A with the linear spaces of 1-forms on ℝ3 and 2-forms on ℝ3. If we choose an orthonormal oriented coordinate system (x1, x2, x3) on ℝ3, then
andRemark. Thus the isomorphisms do not depend on the choice of the orthonormal oriented coordinate system (x1, x2, x3). But they do depend on the choice of the euclidean structure on ℝ3, and the isomorphismalso depends on the orientation (coming implicitly in the definition of triple scalar product).
Problem 6. Show that, under the isomorphisms established above, the exterior product of 1-forms becomes the vector product in ℝ3, i.e., thatIn this way the exterior product of 1-forms can be considered as an extension of the vector product in ℝ3 to higher dimensions. However, in the n-dimensional case, the product is not a vector in the same space: the space of 2-forms on ℝn is isomorphic to ℝn only for n = 3.Problem 7. Show that, under the isomorphisms established above, the exterior product of a 1-form and a 2-form becomes the scalar product of vectors in ℝ3:
Problem 8. Verify that f*ωk is an exterior form.Problem 9. Verify that f* is a linear operator from the space of k-forms on ℝn to the space of k-forms on ℝm (the star superscript means that f* acts in the opposite direction from f).Problem 10. Let f: ℝm → ℝn and g: ℝn → ℝp. Verify that (g ○ f)* = f* ○ g*.Problem 11. Verify that f* preserves exterior multiplication: f*(ωk ⋀ ωl) = (f*ωk) ⋀(f*ωl)
We give here the definition of differential forms on differentiable manifolds.
Example. Consider the function y = f(x) = x2. Its differential df = 2x dx depends on the point x and on the “increment of the argument,” i.e., on the tangent vector ξ to the x axis. We fix the point x. Then the differential of the function at x, df |x, depends linearly on ξ. So, if x = 1 and the coordinate of the tangent vector ξ is equal to 1, then df = 2, and if the coordinate of ξ is equal to 10, then df = 20 (Figure 140).
Figure 140Differential of a function
Problem 1. Let ξ be the velocity vector of the plane curve x(t) = cos t, y(t) = sin t at t = 0. Calculate the values of the differentials dx and dy of the functions x and y on the vector ξ (Figure 141).Answer.
Problem 2. Show that every differential 1-form on the line is the differential of some function.Problem 3. Find differential 1-forms on the circle and the plane which are not the differential of any function.
Problem 4. Calculate the value of the form ω1 = dx1, w2 = x1dx2, andon the vectors ξ1, ξ2, and ξ3 (Figure 142).
Answer.
Problem 5. Let x1,..., xn be functions on a manifold M forming a local coordinate system in some region. Show that every 1-form on this region can be uniquely written in the form ω = a(x) dx1 + ⋯ + an(x) dxn.
Problem 6. Put a natural differentiable manifold structure on the set whose elements are k-tuples of vectors tangent to M at some point x.
Problem 7. Show that the k-forms on M form a vector space (infinite-dimensional if k does not exceed the dimension of M).
Problem 8. Calculate the value of the forms ω1 = dx1 ⋀ dx2, ω2 = x1 dx1 ⋀ dx2 − x2 dx2 ⋀ dx1 and ω3 = r dr ⋀ dφ (where x1 = r cos φ and x2 = r sin φ) on the pairs of vectors (ξ1, η1), (ξ2, η2), and (ξ3, η3) (Figure 143).
Figure 143Problem 8Answer.Problem 9. Calculate the value of the forms ω1 = dx2 ⋀ dx3, ω2 = x1 dx3 ⋀ dx2, and, on the pair of vectors ξ = (1, 1, 1), η = (1, 2, 3) at the point x = (2, 0, 0).
Answer. ω1 = 1, ω2 = −2, ω3 = −8.Problem 10. Let x1,..., xn: M → ℝ be functions on a manifold which form a local coordinate system on some region. Show that every differential form on this region can be written uniquely in the formExample. Change of variables in a form. Suppose that we are given two coordinate systems on ℝ3: x1, x2, x3 and y1, y2, y3. Let ω be a 2-form on ℝ3. Then, by the theorem above, ω can be written in the system of x-coordinates as ω = X1 dx2 ⋀ dx3 + X2 dx3 ⋀ dx1 + X3 dx1 ⋀ dx2, where X1, X2, and X3 are functions of x1, x2, and x3, and in the system of y-coordinates as ω = Y1 dy2 ⋀ dy3 + Y2 dy3 ⋀ dy1 + Y3 dy1 ⋀ dy2, where Y1, Y2, and Y3 are functions of y1, y2, and y3.Problem 11. Given the form written in the x-coordinates (i.e., the Xi) and the change of variables formulas x = x(y), write the form in y-coordinates, i.e., find Y.Solution. We have dxi = (∂xi/∂y1) dy1 + (∂xi/ay2) dy2 + (∂xi/∂y3) dy3 . Therefore,from which we get
Problem 12. Find E1, E2, and E3 for cartesian coordinates x, y, z, for cylindrical coordinates r, φ, z and for spherical coordinates R, φ, θ in the euclidean space ℝ3 (Figure 144).Answer.
Problem 13. Find the values of the forms dx1; dx2, and dx3 on the vectors e1, e2, and e3.Answer., the rest are zero. In particular, for cartesian coordinates dx(ex) = dy(ey) = dz(ez) = 1; for cylindrical coordinates dr(er) = dz(ez) = 1 and dφ(eφ) = 1/r (Figure 145), for spherical coordinates dR(eR) = 1, dφ(eφ) = 1/R cos θ and dθ(eθ) = 1/R.
Figure 145Problem 13
Problem 14. Calculate [e1, e2], (eR, eθ), and (ez, ex, ey).Answer. e3, 0, 1.
Problem 15. Given the components of the vector field A, find the decompositions of the 1-formand the 2-form
.
Problem 16. Find the components of the gradient of a function in the basis e1, e2, e3.
We define here the concepts of a chain, the boundary of a chain, and the integration of a form over a chain.The integral of a differential form is a higher-dimensional generalization of such ideas as the flux of a fluid across a surface or the work of a force along a path.
Problem 1. Show that ∫D ωk depends linearly on ωk.Problem 2. Show that if we divide D into two distinct polyhedra D1 and D2, then
Example. Ifand ω = dy, then
Problem 3. Show that f*ω is a k-form on M.Problem 4. Show that the map f* preserves operations on forms:Problem 5. Let g : L → M be a differentiable map. Show that (fg)* = g*f*.Problem 6. Let D1 and D2 be two compact, convex polyhedra in the oriented k-dimensional space ℝk and f: D1 → D2 a differentiable map which is an orientation-preserving diffeomorphism55 of the interior of D1 onto the interior of D2. Then, for any differential k-form ωk on D2,Hint. This is the change of variables theorem for a multiple integral:
Problem 7. Show that the integral depends linearly on the form:
Problem 8. Show that, under a change of orientation, the integral changes sign:
Problem 9. Show that the set of all k-chains on M forms a commutative group if we define the addition of chains by the formula
Problem 10. Show that the boundary of the boundary of any chain is zero: ∂∂ck = 0.Hint. By the linearity of ∂ it is enough to show that ∂∂D = 0 for a convex polyhedron D. It remains to verify that every (k − 2)-dimensional face of D appears in ∂∂D twice, with opposite signs. It is enough to prove this for k = 2 (planar cross-sections).
Problem 11. Show that the integral depends linearly on the form:Problem 12. Show that integration of a fixed form ωk on chains ck defines a homomorphism from the group of chains to the line.Example 1. Let M be the plane {(p, q)}, ω1 the form pdq, and c1 the chain consisting of one cell σ with multiplicity 1:Then ∫c1 pdq = π. In general, if a chain c1 represents the boundary of a region G (Figure 154), then ∫c1 pdq is equal to the area of G with sign + or − depending on whether the pair of vectors (outward normal, oriented boundary vector) has the same or opposite orientation as the pair (p axis, q axis).
Figure 154The integral of the form p dq over the boundary of a region is equal to the area of the region.Example 2. Let M be the oriented three-dimensional euclidean space ℝ3. Then every 1-form on M corresponds to some vector field, where
The integral ofon a chain c1 representing a curve l is called the circulation of the field A over the curve l:
Every 2-form on M also corresponds to some field. The integral of the form
on a chain c2 representing an oriented surface S is called the flux of the field A through the surface S:
Problem 13. Find the flux of the field A = (1/R2)eR over the surface of the sphere x2 + y2 + z2 = 1, oriented by the vectors ex, ey at the point z = 1. Find the flux of the same field over the surface of the ellipsoid (x2/a2) + (y2/b2) + z2 = 1 oriented the same way.Hint. Cf. Section 36H.Problem 14. Suppose that, in the 2n-dimensional space ℝn = {(p1,...,pn; q1,...,qn)}, we are given a 2-chain c2 representing a two-dimensional oriented surface S with boundary l. FindAnswer. The sum of the oriented areas of the projection of S on the two-dimensional coordinate planes pi, qi.
Example. We will call a smooth function φ: M → R a 0-form on M. The integral of the 0-form φ on the 0-chain c0 = ∑ miAi (where the mi are integers and the Ai points of M) isThen the definition above gives the “increment” F(ξ1) = φ(x1) − φ(x) (Figure 157) of the function φ, and the principal linear part of F(ξ1) at 0 is simply the differential of φ.
Figure 157The integral over the boundary of a one-dimensional parallelepiped is the change in the function.Problem 1. Show that the function F(ξ1,...,ξk + 1) is skew-symmetric with respect to ξ.
Problem 3. Prove the formulas for differentiating a sum and a product:andProblem 4. Show that the differential of a differential is equal to zero: dd = 0.Problem 5. Let f: M → N be a smooth map and ω a k-form on N. Show that f*(dω) = d(f*ω).
Problem 6. Show thatHint. By the formula for differentiating the product of forms,Problem 7. Show that curl grad = div curl = 0.Hint. dd = 0.
Problem 8. Given the components of a vector field A = A1e1 + A2e2 + A3e3, find the components of its curl.Solution. According to Section 34ETherefore,According to Section 34E, we haveIn particular, in cartesian, cylindrical, and spherical coordinates on ℝ3,Problem 9. Find the divergence of the field A = A1e1 + A2e2 + A3e3.Solution.. Therefore,
By the definition of divergence,This meansIn particular, in cartesian, cylindrical, and spherical coordinates on ℝ3:Problem 10. The Laplace operator on M is the operator Λ = div grad. Find its expression in the coordinates xi.Answer.In particular, on ℝ3
Problem 12. Prove Poincaré’s lemma for 1-forms.Hint. Consider.
Problem 13. Show that in a vector space the integral of a closed form over any cycle is zero.Hint. Construct a (k + 1)-chain whose boundary is the given cycle (Figure 164).
Figure 164Cone over a cycleNamely, for any chain c consider the “cone over c with vertex 0.” If we denote the operation of constructing a cone by p, thenTherefore, if the chain c is closed, ∂(pc) = c.Problem. Show that every closed form on a vector space is an exterior derivative.Hint. Use the cone construction. Let ωk be a differential k-form on ℝn. We define a (k − 1)-form (the “co-cone over ω”) pωk in the following way: for any chain ck − 1It is easy to see that the (k − 1)-form pωk exists and is unique; its value on the vectors ξ1,...,ξk − 1, tangent to ℝn at x, is equal toIt is easy to see thatTherefore, if the form ωk is closed, d(pωk) = ωk.Problem. Let X be a vector field on M and ω a differential k-form. We define a differential (k − 1)-form ixω (the interior derivative of ω by X) by the relationProve the homotopy formulawhere Lx is the differentiation operator in the direction of the field X.[The action of Lx on a form is defined, using the phase flow {gt} of the field X, by the relationLX is called the Lie derivative or fisherman’s derivative: the flow carries all possible differential-geometric objects past the fisherman, and the fisherman sits there and differentiates them.]Hint. We denote by H the “homotopy operator” associating to a k-chain γ: σ → M the (k + 1)-chain Hγ: (I × σ) → M according to the formula (Hγ)(t, x) = gtγ(x) (where I = [0, 1]). ThenProblem. Prove the formula for differentiating a vector product on three-dimensional euclidean space (or on a riemannian manifold):(where {a, b} = Lab is the Poisson bracket of the vector fields, cf. Section 39).Hint. If τ is the volume element, thenby using these relations and the fact that dτ = 0, it is easy to derive the formula for curl[a, b] from the homotopy formula.
The dimension of the space Hk(M, ℝ) is called the k-th Betti number of M.Problem 14. Show that for the circle S1 we have H1(S1, ℝ) = ℝ.
Problem 15. Find the first Betti number of the torus T2 = S1 × S1.
We define here symplectic manifolds, hamiltonian vector fields, and the standard symplectic structure on the cotangent bundle.
Problem. Verify that (ℝ2n, ω2) is a symplectic manifold. For n = 1 the pair (ℝ2, ω2) is the pair (the plane, area).
Remark. Consider a lagrangian mechanical system with configuration manifold V and lagrangian function L. It is easy to see that the lagrangian “generalized velocity”is a tangent vector to the configuration manifold V, and the “generalized momentum”
is a cotangent vector. Therefore, the “p, q” phase space of the lagrangian system is the cotangent bundle of the configuration manifold. The theorem above shows that the phase space of a mechanical problem has a natural symplectic manifold structure.
Problem. Show that the correspondenceis an isomorphism between the 2n-dimensional vector spaces of vectors and of 1-forms.
Example. In ℝ2n = {(p, q)} we will identify vectors and 1-forms by using the euclidean structure (x, x) = p2 + q2. Then the correspondencedetermines a transformation ℝ2n → ℝ2n.
Problem. Calculate the matrix of this transformation in the basis p, q.
Answer.
Liouville’s theorem asserts that the phase flow preserves volume. Poincaré found a whole series of differential forms which are preserved by the hamiltonian phase flow.
Definition. The vector field I dH is called a hamiltonian vector field; H is called the hamiltonian function.
Problem. Is every one-parameter group of diffeomorphisms of M2n which preserves the symplectic structure a hamiltonian phase flow?Hint. Cf. Section 40.
Example. If M = ℝ2 and ω2 = dp ⋀ dq is the area element, then ω2 is an integral invariant of any map g with jacobian 1.Problem. Show that a form ωk is an integral invariant of a map g if and only if g*ωk = ωk.Problem. Show that if the forms ωk and ωt are integral invariants of the map g, then the form ωk ⋀ ωt is also an integral invariant of g.
Problem. Suppose that the dimension of the symplectic manifold (M2n, ω2) is 2n. Show that (ω2)k = 0 for k > n, and that (ω2)n is a nondegenerate 2n-form on M2n.
Example. A canonical map g: ℝ2n → ℝ2n has the 1-formIn fact, every closed chain c on ℝ2n is the boundary of some chain σ, and we find(1 and 6 are by definition of σ, 2 by definition of ∂, 3 and 5 by Stokes’ formula, and 4 since g is canonical and dω1 = d(p dq) = dq ⋀ dq = ω2).
Problem. Let dωk be an absolute integral invariant of the map g: M → M. Does it follow that ωk is a relative integral invariant?
Answer. No, if there is a closed k-chain on M which is not a boundary.
Problem. Show that the 1-form dH is an integral invariant of the phase flow with hamiltonian function H.
Every pair of vector fields on a manifold determines a new vector field, called their Poisson bracket.60 The Poisson bracket operation makes the vector space of infinitely differentiable vector fields on a manifold into a Lie algebra.
Problem. Show that the set of n × n matrices becomes a Lie algebra if we define the commutator by [A, B] = AB − BA.
Problem. Show that the operator LA is linear:Also, prove Leibniz’s formula LA(φ1φ2) = φ1LAφ2 + φ2LAφ1.Example. Let (x1,..., xn) be local coordinates on M. In this coordinate system the vector A(x) is given by its components (A1(x),..., An(x)); the flow At is given by the system of differential equationsand, therefore, the derivative of φ = φ(x1,..., xn) in the direction A isWe could say that in the coordinates (x1,..., xn) the operator LA has the formthis is the general form of a first-order linear differential operator on coordinate space.Problem. Show that the correspondences between vector fields A, flows At, and differentiations LA are one-to-one.
Problem. Find an example.Solution. The fields A = e1, B = x1e2 on the (x1, x2) plane.
Problem. Suppose that the vector fields A and B are given by their components Ai, Bi in coordinates xi. Find the components of the Poisson bracket.
Problem. Let A1 be the linear vector field of velocities of a rigid body rotating with angular velocity ω1 around 0, and A2 the same thing with angular velocity ω2 . Find the Poisson bracket [A1, A2].
Consider the rectangle 0 ≤ t ≤ t0, 0 ≤ s ≤ s0 (Figure 170) in the t, s-plane. To every path going from (0, 0) to (t0, s0) and consisting of a finite number of intervals in the coordinate directions, we associate a product of transformations of the flows At and Bs. Namely, to each interval t1 ≤ t ≤ t2 we associate At2−t1, and to each interval s1 ≤ s ≤ s2 we associate Bs2−s1; the transformations are applied in the order in which the intervals occur in the path, beginning at (0, 0). For example, the sides (0 ≤ t ≤ t0, s = 0) and (t = t0, 0 ≤ s ≤ s0) corresponds to the product Bs0At0, and the sides (t = 0, 0 ≤ s ≤ s0) and (s = s0, 0 ≤ t ≤ t0) to the product At0Bs0.
Figure 170Proof of the commutativity of flowsIn addition, we associate to each such path in the (t, s)-plane a path on the manifold M starting at the point x and composed of trajectories of the flows At and Bs (Figure 171). If a path in the (t, s)-plane corresponds to the product At1Bs1 ··· AtnBsn, then on the manifold M the corresponding path ends at the point At1Bs1x ··· AtnBsnx. Our goal will be to show that all these paths actually terminate at the one point At0Bs0x = Bs0At0x.
Figure 171Curvilinear quadrilateral βγδεαWe partition the intervals 0 ≤ t ≤ t0 and 0 ≤ s ≤ s0 into N equal parts, so that the whole rectangle is divided into N2 small rectangles. The passage from the sides (0, 0) − (t0, 0) − (t0, s0) to the sides (0, 0) − (0, s0) − (t0, s0) can be accomplished in N2 steps, in each of which a pair of neighboring sides of a small rectangle is exchanged for the other pair (Figure 172). In general, this small rectangle corresponds to a non-closed curvilinear quadrilateral βγδεα on the manifold M (Figure 171). Consider the distance63 between its vertices α and β corresponding to the largest values of s and t. As we saw earlier, ρ(α, β) ≤ C1N−3 (where the constant C1 > 0 does not depend on N). Using the theorem of the differentiability of solutions of differential equations with respect to the initial data, it is not difficult to derive from this a bound on the distance between the ends α′ and β′ of the paths xδγββ′ and xδεαα′ on M: ρ(α′,β′) < C2 N− 3, where the constant C2 > 0 again does not depend on N. But we broke up the whole journey from Bs0At0x to At0Bs0x into N2 such pieces. Thus, ρ(At0Bs0x, Bs0At0x) < N2C2N−3 ∀N. Therefore, At0Bs0x = Bs0At0x. □
Figure 172Going from one pair of sides to the other
Problem. Compute the bracket operation in the Lie algebra of the group SO(3) of rotations in three-dimensional euclidean space.
The hamiltonian vector fields on a symplectic manifold form a subalgebra of the Lie algebra of all fields. The hamiltonian functions also form a Lie algebra: the operation in this algebra is called the Poisson bracket of functions. The first integrals of a hamiltonian phase flow form a subalgebra of the Lie algebra of hamiltonian functions.
Problem 1. Compute the Poisson bracket of two functions F and H in the canonical coordinate space ℝ2n = {(p, q)}, ω2(ξ, η) = (Iξ, η).Solution. By Corollary 3 we have(we use the fact that I is symplectic and has the formin the basis (p, q)).
Problem 2. Compute the Poisson brackets of the basic functions pi and qj.Solution. The gradients of the basic functions form a “symplectic basis”: their skew-scalar products are
Problem 3. Show that the map A: ℝ2n → ℝ2n sending (p, q) → (P(p, q), Q(p, q)) is canonical if and only if the Poisson brackets of any two functions in the variables (p, q) and (P, Q) coincide:Solution. Let A be canonical. Then the symplectic structures dp ∧ dq and dP ∧ dQ coincide. But the definition of the Poisson bracket (F, H) was given invariantly in terms of the symplectic structure; it did not involve the coordinates. Therefore,Conversely, suppose that the Poisson brackets (Pi, Qj)p, q have the standard form of Problem 2. Then, clearly, dP ∧ dQ = dp ∧ dq, i.e., the map A is canonical.Problem 4. Show that the Poisson bracket of a product can be calculated by Leibniz’s rule:Hint. The Poisson bracket (F1F2, H) is the derivative of the product F1F2 in the direction of the field I dH.
Problem. Calculate the Poisson brackets of the components p1, p2, p3, M1, M2, M3 of the linear and angular momentum vectors of a mechanical system.Answer. (M1, M2) = M3, (M1, p1) = 0, (M1, p2) = p3, (M1, p3) = −p2. This impliesTheorem. If two components, M1 and M2, of the angular momentum of some mechanical problem are conserved, then the third component is also conserved.
Let (M2n, ω2) be a symplectic manifold and gt: M2n → M2n a one-parameter group of diffeomorphisms preserving the symplectic structure. Will gt be a hamiltonian flow?Example. Let M2n be a two-dimensional torus T2, a point of which is given by a pair of coordinates (p, q)mod 1. Let ω2 be the usual area element dp ∧ dq. Consider the family of translations gt(p, q) = (p + t, q) (Figure 173). The maps gt preserve the symplectic structure (i.e., area). Can we find a hamiltonian function corresponding to the vector field (= 1,
= 0)? If
and
, we would have ∂H/∂p = 0 and ∂H/∂q = −1, i.e., H = −q + C. But q is only a local coordinate on T2; there is no map H: T2 → ℝ for which ∂H/∂p = 0 and ∂H/∂q = 1. Thus gt is not a hamiltonian phase flow.
Figure 173A locally hamiltonial field on the torus
Definition. A locally hamiltonian vector field on a symplectic manifold (M2n, ω2) is the vector field Iω1, where ω1 is a closed 1-form on M2n.Locally, a closed 1-form is the differential of a function, ω1 = dH. However, in attempting to extend the function H to the whole manifold M2n we may obtain a “many-valued hamiltonian function,” since a closed 1-form on a non-simply-connected manifold may not be a differential (for example, the form dq on T2). A phase flow given by a locally hamiltonian vector field is called a locally hamiltonian flow.Problem. Show that a one-parameter group of diffeomorphisms of a symplectic manifold preserves the symplectic structure if and only if it is a locally hamiltonian phase flow.Hint. Cf. Section 38A.Problem. Show that in the symplectic space ℝ2n, every one-parameter group of canonical diffeomorphisms (preserving dp ⋀ dq) is a hamiltonian flow.Hint. Every closed 1-form on ℝ2n is the differential of a function.Problem. Show that the locally hamiltonian vector fields form a sub-algebra of the Lie algebra of all vector fields. In addition, the Poisson bracket of two locally hamiltonian fields is actually a hamiltonian field, with a hamiltonian function uniquely65 determined by the given fields ξ and η by the formula H = ω2(ξ, η). Thus, the hamiltonian fields form an ideal in the Lie algebra of locally hamiltonian fields.
A euclidean structure on a vector space is given by a symmetric bilinear form, and a symplectic structure by a skew-symmetric one. The geometry of a symplectic space is different from that of a euclidean space, although there are many similarities.
Problem. Show that ξ ∠ ξ: every vector is skew-orthogonal to itself.
Problem. Find the skew-scalar product of the basis vectors epi and eqi (i = 1 ..., n) in the example presented above.Solution. The relationsfollow from the definition of p1 ∧ q1 + ... + pn ∧ qn.(1)
Problem. Show that the group Sp(2) is isomorphic to the group of real two-by-two matrices with determinant 1 and is homeomorphic to the interior of a solid three-dimensional torus.
Problem. Show that a nonzero vector in a symplectic space can be carried into any other nonzero vector by a symplectic transformation.Problem. Show that not every two-dimensional plane of the symplectic space ℝ2n can be obtained from a given 2-plane by a symplectic transformation.Hint. Consider the planes (p1, p2) and (p1, q1).
Example. The coordinate plane (p1,..., pk) in the symplectic coordinate system p, q is null. (Prove it!)Problem. Show that any non-null two-dimensional plane can be carried into any other non-null two-plane by a symplectic transformation.
Problem. Compute the matrix of the operator I in the symplectic basis epi, eqi.Answer.where E is the n × n identity matrix.
Problem. Show that the operator I is symplectic and that I2 = −E2n.
Problem. Show that there are 2n null planes among then-dimensional coordinate planes: to each of the 2n partitions of the set (1,..., n) into two parts (i1,..., ik), (j1,..., jn − k) we associate the null coordinate plane pi1,..., pik, qj1,..., qjn − k.
Problem. Let π1 and π2 be two k-dimensional planes in symplectic ℝ2n. Is it always possible to carry π1 to π2 by a symplectic transformation? How many classes of planes are there which cannot be carried one into another?Answer. [k/2] + 1, if k ≤ n: [(2n − k)/2] + 1 if k ≥ n.
Problem. Show that transformations which are both orthogonal and symplectic are complex, that those which are both complex and orthogonal are symplectic, and that those which are both symplectic and complex are orthogonal; thus that the intersection of two of the three groups is equal to the intersection of all three:This intersection is called the unitary group U(n).
During our investigation of oscillating systems with periodically varying parameters (cf. Section 25), we explained that parametric resonance depends on the behavior of the eigenvalues of a certain linear transformation (“the mapping at a period”). The dependence consists of the fact that an equilibrium position of a system with periodically varying parameters is stable if the eigenvalues of the mapping at a period have modulus less than 1, and unstable if at least one of the eigenvalues has modulus greater than 1.The mapping at a period obtained from a system of Hamilton’s equations with periodic coefficients is symplectic. The investigation in Section 25 of parametric resonance in a system with one degree of freedom relied on our analysis of the behavior of the eigenvalues of symplectic transformations of the plane. In this paragraph we will analyze, in an analogous way, the behavior of the eigenvalues of symplectic transformations in a phase space of any dimension. The results of this analysis (due to M. G. Krein) can be applied to the study of conditions for the appearance of parametric resonance in mechanical systems with many degrees of freedom.
Problem. Show that if at least one of the eigenvalues of a symplectic transformation S does not lie on the unit circle, then S is unstable.Hint. In view of the demonstrated symmetry, if one of the eigenvalues does not lie on the unit circle, then there exists an eigenvalue outside the unit circle |λ| > 1; in the corresponding invariant subspace, S is an “expansion with a rotation.”Problem. Show that if all the eigenvalues of a linear transformation are distinct and lie on the unit circle, then the transformation is stable.Hint. Change to a basis of eigenvectors.
Problem. Let λ andbe simple (multiplicity 1) eigenvalues of a symplectic transformation S with |λ| = 1. Show that the two-dimensional invariant plane πλ corresponding to λ,
, is non-null.
Hint. Let ξ1 and ξ2 be complex eigenvectors of S with eigenvalues λ1 and λ2. Then if λ1 λ2 ≠ 1, the vectors ξ1 and ξ2 are skew-orthogonal: [ξ1, ξ2] = 0.Let ξ be a real vector of the plane πλ, where Im λ > 0 and |λ| = 1. The eigenvalue λ is called positive if [Sξ, ξ] > 0.Problem. Show that this definition is correct, i.e., it does not depend on the choice of ξ ≠ 0 in the plane πλ.Hint. If the plane πλ contained two non-collinear skew-orthogonal vectors, it would be null.In the same way, an eigenvalue λ of multiplicity k with |λ| = 1 is of definite sign if the quadratic form [Sξ, ξ] is (positive or negative) definite on the invariant 2k-dimensional subspace corresponding to λ,.
Problem. Show that S is strongly stable if and only if all the eigenvalues λ lie on the unit circle and are of definite sign.Hint. The quadratic form [Sξ, ξ] is invariant with respect to S.
Problem. Show that a symplectic atlas defines a symplectic structure on M2n.
In this section we look at the geometry of 1-forms in an odd-dimensional space.
Problem. Consider the 2-form ω2 = dp1 ⋀ dq1 + ⋯ + dpn ⋀ dqn on an even-dimensional space ℝ2n with coordinates p1,..., pn; q1,..., qn. Show that ω2 is nonsingular.Problem. On an odd-dimensional space ℝ2n + 1 with coordinates p1,..., pn; q1,..., qn; t, consider the 2-form ω2 = ∑dpi ⋀ dqi − ω1 ⋀ dt, where ω1 is any 1-form onℝ2n + 1 Show that ω2 is nonsingular.
Problem. Show that definitions (1) and (2) are equivalent to (3) if the domain of the map in question is a simply connected region in the phase space ℝ2n; in the general case 3 ⇒ 2 ⇔ 1.
the volume of gD is equal to the volume of D, for any region D.
In this paragraph we prove that canonical transformations preserve the form of Hamilton’s equations, that a first integral of Hamilton’s equations allows us to reduce immediately the order of the system by two and that motion in a natural lagrangian system proceeds along geodesics of the configuration space provided with a certain riemannian metric.
Problem. Let g(t): ℝ2n → ℝ2n be a canonical transformation of phase space depending on the parameter t, g(t)(p, q) = (P(p, q, t), Q(p, q, t)). Show that in the variables P, Q, t the canonical equations (1) have the canonical form with new hamiltonian function
Problem. Show that the curveis a vortex line of the form p dq = P dQ − K dT on M2n−1. Hint. d(Ht) does not affect the vortex lines, and dH is zero on M.
Remark. The principle of least action in Hamilton’s form is a particular case of the principle considered above. Along extremals, we have(since the lagrangian L and the hamiltonian H are Legendre transforms of one another). Now let(Figure 188) be the projection of the extremal γ onto the q, t plane. To any nearby curve
connecting the same points (t0, q0) and (t1, q,) in the q, t plane we associate a curve γ′ in the phase space (p, q, t) by setting
. Then, along γ′, too,
. But by the theorem above, δ ∫γ p dq − H dt = 0 for any variation curve γ(with boundary conditions (t = t0, q = q0) and (t = t1, q = q1). In particular, this is true for variations of the special form taking γ to γ′. Thus γ is an extremal of ∫ L dt, as was to be shown.
The fundamental notions of hamiltonian mechanics (momenta, the hamiltonian function H, the form p dq − H dt and the Hamilton-Jacobi equations, all of which we will be concerned with below) arose by the transforming of several very simple and natural notions of geometric optics, guided by a particular variational principle—that of Fermat, into general variational principles (and in particular into Hamilton’s principle of stationary action, δ ∫ L dt = 0).
Optics
|
Mechanics
|
---|---|
Optical medium
|
Extended configuration space {(q, t)}
|
Fermat’s principle
|
Hamilton’s principle δ ∫ L dt = 0
|
Rays
|
Trajectories q(t)
|
Indicatrices
|
Lagrangian L
|
Normal slowness vector p of the front
|
Momentum p
|
Expression of p in terms of the velocity of the ray,
|
Legendre transformation
|
1-form p dq
|
1-form p dq − H dt
|
By Stokes’ lemma, ∫γ1 − ∫γ2 + ∫β − ∫α p dq − H dt = 0. But on α, H dt = 0 and p = ∂S0/dq, soFurther, γ1 and γ2 are phase trajectories, soSoFor Δt, Δq → 0, we get ∂S/∂t = −H, ∂S/∂q = p, which proves the theorem. ☐Problem. Show the uniqueness of the solution to problem (2).Hint. Differentiate S along the characteristics.Problem. Solve the Cauchy problem (2) forAnswer. Cf. Figure 203.
Figure 203A typical singularity of a solution of the Hamilton-Jacobi equation
In this paragraph we define the generating function of a free canonical transformation.
Problem. Show the converse: if this form is an exact differential, then the transformation is canonical.
Problem. Express the hamiltonian function in elliptic coordinates.Solution. The lines ξ = const are ellipses with foci at O1 and O2; the lines η = const are hyperbolas with the same foci (Figure 205). They are mutually orthogonal; therefore,We will find the coefficients a and b. For motion along an ellipse we have dr1 = ds cos α and dr2 = −ds cos α, so dη = 2 cos α ds. For motion along a hyperbola we have dr1 = ds sin α and dr2 = ds sin α, so dξ = 2 sin α ds. Thus a = (2 sin α) − 1 and b = (2 cos α)− 1. Furthermore, from the triangle O1 MO2 we find, which implies
But if, then
Thus,But r1 + r2 = ξ, r1 − r2 = η, 4r1r2 = ξ2 − η2. Therefore, finally,We will now solve the Hamilton-Jacobi equation.
Figure 205Confocal ellipses and hyperbolas
In this paragraph we construct the apparatus of generating functions for non-free canonical transformations.
Problem. Find a generating function S2 for the identity map P = p, Q = q.Answer. Pq.Remark. The generating function S2(P, q) is convenient also because there are no minus signs in the formulas (2), and they are easy to remember if we remember that the generating function of the identity transformation is Pq.
We consider the differential of our transformation g at the point (p0, q0). By identifying the tangent space to ℝ2n with ℝ2n, we can consider dg as a symplectic transformation S : ℝ2n → ℝ2n.Consider the coordinate p-plane P in ℝ2n (Figure 209). This is a null n-plane, and its image SP is also a null plane. We project the plane SP onto the coordinate plane σ = {(pi, qj)} parallel to the remaining coordinate axes, i.e., in the direction of the n-dimensional null coordinate plane. We denote the projection operator by T S : P → σ.
Figure 209Checking non-degeneracyThe condition det(∂(Pi, Qj)/∂(pi, pj)) ≠ 0 means that T : SP → σ is nonsingular. The operator S is nonsingular. Therefore, TS is nonsingular if and only if T : SP → σ is nonsingular. In other words, the null plane SP must be transverse to the null coordinate plane. But we showed in Section 41 that at least one of the 2n null coordinate planes is transverse to SP. This means that one of our 2n determinants is nonzero, as was to be shown. ☐
Problem. Show that this system of 2n types of generating functions is minimal: given any one of the 2n determinants, there exists a canonical transformation for which only this determinant is nonzero.86
Problem. Show thatexists and does not depend on the representative of the class gε.
In order to integrate a system of 2n ordinary differential equations, we must know 2n first integrals. It turns out that if we are given a canonical system of differential equations, it is often sufficient to know only n first integrals—each of them allows us to reduce the order of the system not just by one, but by two.
Problem 1. Show that the map g (Figure 211) of a sufficiently small neighborhood V of the point 0 ∈ ℝn gives a chart in a neighborhood of x0: every point x0 ∈ M has a neighborhood U (x0 ∈ U ⊂ M) such that g maps V diffeomorphically onto U.
Figure 211Problem 1Hint. Apply the implicit function theorem and use the linear independence of the fields at x0.Problem 2. Show that g: ℝn → M is onto.Hint. Connect a point x ∈ M with x0 by a curve (Figure 212), cover the curve by a finite number of the neighborhoods U of the preceding problem and define t as the sum of shifts ti corresponding to pieces of the curve.
Figure 212Problem 2
Problem 3. Show that Γ is a subgroup of the group ℝn, independent of the point x0.Solution. If gsx0 = x0 and gtx0 = x0, then gs+tx0 = gsgtx0 = gsx0 = x0 and g−tx0 = g−tgtx0 = x0. Therefore, Γ is a subgroup of ℝn. If x = grx0 and t ∈ Γ, then gtx = gt+rx0 = grgtx0 = grx0 = x.
Problem 4. Show that, in a sufficiently small neighborhood V of the point 0 ∈ ℝn, there is no point of the stationary group other than t = 0.Hint. The map g: V → U is a diffeomorphism.Problem 5. Show that, in the neighborhood t + V of any point t ∈ Γ ⊂ ℝn, there is no point of the stationary group Γ other than t. (Figure 213)
Figure 213Problem 5
Problem 6. Show that the distance from this axis to any point e of Γ not lying on ℝe1 is greater than or equal to the distance of e2 from ℝe1.Hint. By a shift of me1 we can move the projection of e onto the axis interval Δ.
Problem 7. Show that there are no points of Γ on the plane ℝe1 + ℝe2 other than integral linear combinations of e1 and e2.Hint. Partition the plane into parallelograms (Figure 216) Δ = {λ1e1 + λ2e2 }, mi ≤ λi < mi + 1. If there were an e ∈ Δ with e ≠ m1e1 + m2e2, then the point e − m1e1 − m2e2 would be closer to ℝe1 than e2.
Figure 216Problem 7
Problem 8. Show that this closest point always exists.Hint. Take the closest of the finite number of points in a “cylinder” C.
Hint. Partition the plane ℝe1 + ⋯ + ℝek into parallelepipeds Δ and show that there cannot be a point of Γ in any Δ. If there is an e ∈ Γ outside the plane ℝe1 + ⋯ + ⋯ + ℝek, the construction is not finished.
Problem 10. Show that the map of charts A: ℝn → ℝn gives a diffeomorphism à : Tk × ℝn−k → Mf,
Problem 11. Show that under the action of the phase flow with hamiltonian H the angular coordinates φ vary uniformly with timeIn other words, motion on the invariant torus Mf is conditionally periodic.Hint. φ = A−1t.
We show here that, under the hypotheses of Liouville’s theorem, we can find symplectic coordinates (I, φ) such that the first integrals F depend only on I, and φ are angular coordinates on the torus Mf.
Problem. Show that the manifold Mf has a neighborhood diffeomorphic to the direct product of the n-dimensional torus Tn and the disc Dn in n-dimensional euclidean space.Hint. Take the functions Fi and the angles φi constructed above as coordinates. In view of the linear independence of the dFi, the functions Fi and φi (i = 1,..., n) give a diffeomorphism of a neighborhood of Mf onto the direct product Tn × Dn.
Solution. In the variables (I, φ), the equations of the flow (2) have the canonical form with hamiltonian function H(I). Therefore, ω(I) = ∂H/∂I; thus if the number of degrees of freedom is n ≥ 2, the functions ω(I) are not arbitrary, but satisfy the symmetry condition ∂ωi/∂Ij = ∂ωj/∂Ii.
Problem. Find S and I for a harmonic oscillator.Answer. If H = ½a2p2 + ½b2q2 (Figure 217), then Mh is the ellipse bounding the area. Thus for a harmonic oscillator the action variable is the ratio of energy to frequency. The angle variable φ is, of course, the phase of oscillation.
Figure 217Action variable for a hamonic oscillatorProblem. Show that the period T of motion along the closed curve H = h on the phase plane p, q is equal to the derivative with respect to h of the area bounded by this curve:
Problem. Show that this integral does not depend on the choice of the curve γi representing the cycle (Figure 218).
Figure 218Independence of the curve of integration for the action variableHint. In Section 49 we showed that the 2-form ω2 = ∑dpi ⋀ dqi on the manifold Mf is equal to zero. By Stokes’ formula,where ∂σ = γ − γ′.
In this paragraph we show that time averages and space averages are equal for systems undergoing conditionally periodic motion.
Example. Let n = 2. If ω1/ω2 = k1/k2, the trajectories are closed: if (ω1/ω2 is irrational, then trajectories on the torus are dense (cf. Section 16).
Corollary. In the sequenceof first digits of the numbers 2n, the number 7 appears (log 8 − log 7)/(log 9 − log 8) times as often as 8.
Problem. A two-dimensional oscillator with kinetic energyand potential energy U = ½x2 + y2 performs an oscillation with amplitudes ax = 1 and ay = 1. Find the time average of the kinetic energy.
Answer. (ω1α1 + ω2α2 + ω3α3)/π, where α1, α2, and α3 are the angles of the triangle with sides ak (Figure 223).
Figure 223Problem on mean motion of perihelia
Problem. Show that the set of all relations between a given set of frequencies ω is a subgroup Γ of the lattice ℤn.
Problem. Show that, if a system is nondegenerate, then in any neighborhood of any point there is a conditionally periodic motion with n frequencies, and also with any smaller number of frequencies.Hint. We can take the frequencies ω themselves instead of the variables I as local coordinates. In the space of collections of frequencies, the set of points ω with any number of relations r(0 ≤ r < n) is dense.
Problem. Show that the set of I for which the frequencies ω(I) in a nondegenerate system are dependent has Lebesgue measure equal to zero.Hint. Show first that
Here we show the adiabatic invariance of the action variable in a system with one degree of freedom.
To find an accurate estimate, we introduce the quantityThen (6′) and (9) imply(10)
where |R′| < c2ε2 + c5ε|z| if the segment (P, J) lies in G − α. Under this assumption we find(11)
Proof. |z(t)| is no greater than the solution y(t) of the equation. Solving this equation, we find
. □
Now from (11) and the assumption that the segment (P, J) lies in G − α (Figure 226), we haveFrom this it follows that, for 0 ≤ t ≤ 1/cε,We see that, if α = d/3 and ε is small enough, the entire segment (P(t), J(t)) (t ≤ 1/ε) lies inside G − α and, therefore,On the other hand, |P(t) − I(t)| < |εk| < c3 ε. Thus, for all t with 0 ≤ t ≤ 1/ε,and the theorem is proved. □
Problem. Show that ∂S(I, q; λ)/∂λ is a single-valued function on the phase plane.Hint. S is determined up to the addition of multiples of 2πI.
Problem. Given a vector tangent to the sphere at one vertex of a spherical triangle with three right angles, translate this vector around the triangle and back to the same vertex.Answer. As a result of this translation the tangent plane to the sphere at the initial vertex will be turned by a right angle.
Problem. Translate a vector directed towards the North Pole and located at Leningrad (latitude λ = 60°) around the 60th parallel and back to Leningrad, moving to the east.Answer. The vector turns through the angle 2π (1 − sin λ), i.e., approximately 50° to the west. Thus the size of the angle of rotation is proportional to the area bounded by our parallel, and the direction of rotation coincides with the direction the origin of the vector is going around the North Pole.Hint. It is sufficient to translate the vector along the same circle on the cone formed by the tangent lines to the meridian, going through all the points of the parallel (Figure 231). This cone then can be unrolled onto the plane, after which parallel translation on its surface becomes ordinary parallel translation on the plane.
Figure 231Parallel translation on the sphereExample. We consider the upper half-plane y > 0 of the plane of complex numbers z = x + iy with the metricIt is easy to compute that the geodesics of this two-dimensional riemannian manifold are circles and straight lines perpendicular to the x-axis. Linear fractional transformations with real coefficientsare isometric transformations of our manifold, which is called the Lobachevsky plane.Problem. Translate a vector directed along the imaginary axis at the point z = i to the point z = t + i along the horizontal line (dy = 0) (Figure 232).
Figure 232Parallel translation on the Lobachevsky planeAnswer. Under translation by t the vector turns t radians in the direction from the y-axis towards the x-axis.
Problem. Find the curvature forms on the plane, on a sphere of radius R, and on the Lobachevsky plane.
Answer. Ω = 0, Ω = R−2 dS, Ω = −dS, where the 2-form dS is the area element on our oriented surface.
Problem. Show that the function defined by formula (2) is really a differential 2-form, independent of the arbitrary choice involved in the construction, and that the rotation of a vector under translation along the boundary of a finite oriented region D is expressed, in terms of this form, by formula (1).
Problem. Show that the integral of the curvature form over any convex surface in three-dimensional euclidean space is equal to 4π.
Problem. Calculate the riemannian curvature of the euclidean space, the sphere of radius R, and the Lobachevsky plane.Problem. Show that the riemannian curvature does not depend on the orientation of the manifold, but only on its metric.Hint. The 2-forms Ω and dS both change sign under a change of orientation.Problem. Show that, for surfaces in ordinary three-dimensional euclidean space, the riemannian curvature at every point is equal to the product of the inverses of the principal radii of curvature (with minus sign if the centers of curvature lie on opposite sides of the surface).
Problem. Show that parallel translation of vectors from one point of a riemannian manifold to another along a fixed path is a linear isometric operator from the tangent space at the first point to the tangent space at the second point.Problem. Parallel translate any vector along the linein a Lobachevsky space with metricAnswer. Vectors in the directions of the x1 and y axes are rotated by angle τ in the plane spanned by them (rotation is in the direction from the y-axis towards the x1-axis); vectors in the x2-direction are carried parallel to themselves in the sense of the euclidean metric.
In the two-dimensional case we characterized this rotation by one number—the angle of rotation φ. In higher dimensions a skew-symmetric operator plays the role of φ. Namely, any orthogonal operator A which is close to the identity can be written in a natural way in the formwhere Φ is a small skew-symmetric operator.Problem. Compute Φ if A is a rotation of the plane through a small angle φ.Answer.Unlike in the two-dimensional case, the function Φ is not generally additive (since the orthogonal group of n-space for n > 2 is not commutative). Nevertheless, we can construct a curvature form using Φ, describing the “infinitely small rotation caused by parallel translation around an infinitely small parallelogram” in the same way as in the two-dimensional case, i.e., using formula (2).
Problem. Show that the function Ω is a differential 2-form (with values in the skew-symmetric operators on TMx) and does not depend on the choice of coordinates we used to identify TMx and M.
Problem. Find the curvatures of a three-dimensional sphere of radius R and of Lobachevsky space in all possible two-dimensional directions.Answer. R−2, −1.
Theorem. The curvature of a riemannian manifold in the two-dimensional direction determined by a pair of orthogonal vectors ξ, η of length 1 can be expressed in terms of the curvature tensor Ω by the formulawhere the brackets denote the scalar product giving the riemannian metric.(3)
Problem 1. Prove the following properties of covariant differentiation:1.∇ξv is a bilinear function of ξ and v.2.∇ξ fv = (Lξ f)v + f(x)∇ξ v, where f is a smooth function and Lξ f is the derivative of f in the direction of the vector ξ inTMx.3.Lξ〈v, w〉 = 〈∇ξv, w(x)〉 + 〈v(x), ∇ξw〉.4.∇v(x) w − ∇w(x) v = [w, v](x) (whereProblem 2. Show that the curvature tensor can be expressed in terms of covariant differentiation in the following way:where ξ, η, ζ are any vector fields whose values at the point under consideration are ξ0, η0, and ζ0.Problem 3. Show that the curvature tensor satisfies the following identities:Problem 4. Suppose that the riemannian metric is given in local coordinates x1,..., xn by the symmetric matrix gij:Denote by e1,..., en the coordinate vector fields (so that differentiation in the direction ei is ∂i = ∂/∂xi). Then covariant derivatives can be calculated using the formulas in Problem 1 and the following formulas:where (glk) is the inverse matrix to (gkl).By using the expression for the curvature tensor in terms of the connection in Problem 2, we also obtain an explicit formula for the curvature. The numbers Rjjkl = 〈Ω(ej, ej)ek, el〉 are called the components of the curvature tensor.
Problem. Prove the theorem above.Problem. Let M be a surface, y(t) the magnitude of the component of the vector ξ(t) in the direction normal to a given geodesic, and let the length of the vector v(t) be equal to 1. Show that y satisfies the differential equationwhere K = K(t) is the riemannian curvature at the point x(t)(5)
Problem. Using Equation (5), compare the behavior of geodesics close to a given one on the sphere (K = +R−2) and on the Lobachevsky plane (K = −1).
If the potential energy of the newtonian equation we obtained did not depend on time, our conclusion would be rigorous. Let us assume further that the curvature in the different directions containing v is in the intervalThen solutions to the Jacobi equation for normal divergence will be linear combinations of exponential curves with exponent ±λi, where the positive numbers λi are between a and b. Therefore, every solution to the Jacobi equation grows at least as fast as eb|t| as either t → + ∞ or t → − ∞; most solutions grow even faster, with rate ea|t|.
An element g of the group G corresponds to a position of the body obtained by the motion g from some initial state (corresponding to the identity element of the group and chosen abritrarily). Let ω be an element of the algebra.Let eωt be a one-parameter group of rotations with angular velocity ω; ω is the tangent vector to this one-parameter group at the identity. Now we look at the displacementobtained from the displacement g by a rotation with angular velocity ω after a small time τ. If the vector ġ coincides with the vectorthen ω is called the angular velocity relative to space and is denoted by ωs. Thus ωs is obtained from ġ by right translation. In an analogous way we can show that the angular velocity in the body is the left translate of the vector ġ in the algebra.
A factorization of a phase flow gt on a manifold X over a phase flow ft on a manifold Y is a smooth mapping π of X onto Y under which motions gt are mapped to motions ft, so that the following diagram commutes (i.e., πgt = ftπ):In our case, X = T*G is the phase space of the body,is the space of angular momenta. The projection
is defined by left translation
is the phase flow of the body under consideration on the 2n-dimensional space T*G, and ft is the phase flow of the Euler equation in the n-dimensional space of angular momenta
.
Proof. Every vector ξ tangent to V at a point M has the form ξ = {f, M}. where. In particular, the vector field on the right side of Euler’s equation can be written in the form X = {dT, M} (here the differential of the function T at a point M of the vector space
is considered as a vector of the dual space to
, i.e., as an element of the Lie algebra
). It follows from the definitions of the symplectic structure Ω and the operation { , } (cf. subsection A) that for every vector ξ tangent to V at M,
Indeed, suppose that after time t the flow of the fluid gives a diffeomorphism gt, and that the velocity at this moment of time is given by the vector field v. Then the diffeomorphism realized by the flow after time t + τ (where τ is small) will be evτgt up to a quantity small in comparison with τ (here evτ is the one-parameter group with vector v, i.e., the phase flow of the differential equation given by the field v). Therefore, the field of velocities v is obtained from the vector ġ tangent to the group at the point g by right translation. This also implies the right-invariance of the kinetic energy, which is by definition equal to(we assume the density of the fluid to be 1).
Strictly speaking, an infinite-dimensional group of diffeomorphisms is not a manifold. Therefore the exact formulation of the definition above requires additional work: we must choose suitable functional spaces, prove a theorem on existence and uniqueness of solutions, etc. Up to now this has been done only in the case when the dimension of the region of the flow D is equal to 2. However, we will proceed as if these difficulties connected with infinite dimensions did not exist. Thus the following arguments are heuristic in character. It turns out that many of the results can be proved rigorously, independently of the theory of infinite-dimensional manifolds.
Example 2. Consider the planar-parallel flow on the torusWe emphasize that in Theorem 13 it is not a question of stability “in a linear approximation,” but of actual strict Liapunov stability (i.e., with respect to finite perturbations in the nonlinear problem). The difference between these two forms of stability is substantial in this case, since our problem has a hamiltonian character (cf. Theorem 4); for hamiltonian systems asymptotic stability is impossible, so stability in a linear approximation is always neutral and insufficient for a conclusion about the stability of an equilibrium position of the nonlinear problem.
We should note that in general an indefinite quadratic form d2H does not imply instability of the corresponding flow. In general, an equilibrium position of a hamiltonian system can be stable even though the hamiltonian function at this position is neither a maximum nor a minimum. The quadratic hamiltonianis the simplest example of this kind.
It should be emphasized that instability of a flow of an ideal fluid is here understood differently than in section K; it is a question of exponential instability of the motion of the fluid, not of its velocity field. It is possible for a stationary flow to be a Liapunov stable solution of Euler’s equation while the corresponding motion of the fluid is exponentially unstable. The reason is that a small change in the velocity field of a fluid can induce an exponentially growing change in the motion of the fluid. In such a case (stability of the solution of Euler’s equation and negative curvature of the group) we can predict the velocity field, but we cannot predict the motion of the fluid mass without a great loss of accuracy.
Recall that a hermitian scalar product (or hermitian structure) on a complex vector space is a complex linear function on pairs of vectors, which (1) is linear in the first and anti-linear in the second variable, (2) changes its value to the complex conjugate when the arguments are interchanged, and (3) becomes a positive-definite real quadratic form if we take the arguments equal:for ξ ≠ 0.An example of a hermitian scalar product iswhere ξk and ηk are the coordinates of the vectors ξ and η in some basis.(1)
A basis for which a hermitian scalar product has the form (1) always exists, and is called a hermitian-orthonormal basis.The real and imaginary parts of a hermitian scalar product are real bilinear forms. The first is symmetric, and the second skew-symmetric, and both are nondegenerate:The quadratic form (ξ, ξ) is positive-definite.Thus a hermitian structure 〈 , 〉 on a complex vector space gives it a euclidean structure ( , ) and a symplectic structure [ , ]. These two structures are related to the complex structure by the relation
Proof. Assume first that the point z lies on the unit sphere S2n + 1.Decompose the vector ξ into two components: one in the complex line determined by the vector z and the other in the hermitian-orthogonal direction. Note that hermitian-orthogonal to the vector z means euclidean-orthogonal to the vectors z and iz. The vector z is a euclidean normal vector to the sphere S2n + 1 at z. The vector iz is a vector tangent to the circle in which the sphere intersects the complex line passing through z. Thus the component η of the vector ξ which is hermitian-orthogonal to z is tangent to the sphere S2n + 1 and euclidean-orthogonal to the circle in which the sphere intersects the line pz.By the definition of the metric on ℂPn, the riemannian square of the length of the vector is equal to the euclidean square length of the component η of ξ which is hermitian-orthogonal to z.We calculate the component η of ξ, hermitian-orthogonal to z. We write our decomposition asBy hermitian multiplication with z, we findsoCalculating the hermitian square of the vector η, we find 〈η, η〉 = 〈η, ξ〉 andThus, formula (2) is proved for points z of the unit sphere. The general case follows from looking at the homothetic transformation z → z/|z|. □
Proof. We need only verify that the form Ω is closed.Consider the exterior derivative dΩ of the form Ω. This differential 3-form on ℂPn is invariant with respect to mappings induced by unitary transformations of the space ℂn + 1. It follows from this that it is equal to zero.To see this, we look at a hermitian-orthonormal basis e1, ..., en of the tangent space to ℂPn at some point z. Then the vectors e1, ..., en, ie1, ..., ien form a euclidean-orthonormal ℝ-basis. We will show that the value of the form dΩ on any triple of these ℝ-basis vectors is equal to zero. (We assume that n > 1; for n = 1 there is nothing to prove.)Note that in any triple of ℝ-basis vectors at least one is hermitian-orthogonal to the two others. Denote this vector by e. It is easy to construct a unitary transformation of the space ℂn + 1 inducing a motion on ℂPn which fixes the point z and the hermitian-orthogonal complement to e, and changes the direction of e.The value of the form dΩ on our three vectors, e, f, and g is equal to its value on the triple −e, f, and g by the invariance of the form Ω, and is hence equal to zero. □
One other method of constructing a symplectic structure on ℂPn uses the fact that this space may be represented as one of the orbits of the co-adjoint representation of a Lie group, and on every such orbit there is always a standard symplectic structure (cf. Appendix 2, Section A). For the Lie group we can take the group of unitary (preserving the hermitian metric) operators in an (n + 1)-dimensional complex space. The orbits of the co-adjoint representation in this case are the same as of the adjoint representation. In the adjoint representation the operator of reflection through a hyperplane (which changes the sign of the first coordinate and leaves the others fixed) has ℂPn as its orbit, since the reflection operator is uniquely determined by the complex line orthogonal to the hyperplane.
Problem 1. Calculate the symplectic structure Ω in the affine chart w = z1 : z0 of the projective line ℂP1.Answer. Ω = (1/π)(dx ⋀ dy)/(1 + x2 + y2)2, where w = x + iy. The coefficient in the definition of the form Ω is chosen to obtain the usual orientation of the complex line (dx ⋀ dy) and so that the integral of the form Ω along the whole projective line is equal to 1.Problem 2. Show that the symplectic structure Ω in the affine chartof the projective space ℂPn = {(z0 : z1 :...: zn)} is given by the formula
By convention, w0 = 1.Remark. Differential forms on a complex space with complex values (such as dwk and) are defined as complex linear functions of tangent vectors; if wk = xk + iyk, then
The space of such forms in ℂn has complex dimension 2n; the 2n formsfor example, form a ℂ-basis, or the 2n forms dxk, dyk.
Exterior multiplication is defined in the usual way and obeys the usual rules. For example,Let f be a real-smooth function on ℂn (with complex values, in general). An example of such a function is. The differential of the function f is a complex 1-form. Therefore, it can be decomposed in the basis
. The coefficients of this decomposition are called the partial derivatives “with respect to wk” and “with respect to
”:
In calculating exterior derivatives it is also convenient to separate into differentiation d′ with respect to the variable w and d″ with respect to the variable, so that d = d′ + d″.
For example, for a function fFor the differential 1-formthe operators d′ and d″ are defined analogously:Problem 3. Show that the symplectic structure Ω on the affine chartof the projective space ℂPn is given by the formula
Proof. We will show that the difference in the heights of the two points obtained as a result of our two motions along the sides of the parallelogram is the same as the integral of the 1-form ω over the four sides of the parallelogram, up to a quantity small of third order with respect to the sides of the parallelogram.To this end we note that the height of the rise of an integral curve along any path of length ε emanating from the origin has order ε2, since at the origin the plane of the field is horizontal. Therefore, the integrals of the 2-form dω over all four vertical areas over the sides of the parallelogram bounded by the integral curves and the horizontal plane, have order ε3 if the sides are of order ε.The integrals of the form ω along integral curves are exactly equal to zero. Therefore, by Stokes’ formula, the increase in height along the integral curve lying over any of the sides of the parallelogram is equal to the integral of the 1-form ω along this side up to a quantity of third-order smallness.Now the theorem follows directly from the definition of exterior differentiation. □
Proof. The first assertion of the theorem is clear by the construction of the 2-form. The proof of the second assertion can be carried out by exactly the same reasoning we used to prove the commutativity of phase flows for which the Poisson bracket of the velocity fields was equal to zero. We can simply refer to this commutativity, applying it to the integral curves arising over the lines of the coordinate directions in the horizontal plane. □
Proof. We consider the value of the 3-form above on any three distinct coordinate vectors. Only one of these vectors can be the vertical. Therefore, of all the terms entering into the definition of the value of the exterior product of the three vectors, only one is nonzero: the product of the value of the form ω on the vertical vector with the value of the form dω on the pair of horizontal vectors. If the field given by the form is integrable, then the second factor is zero, so our 3-form is zero on arbitrary triples of vectors.Conversely, if the 3-form is equal to zero for any vectors, then it is equal to zero for any triple of coordinate vectors, of which one is vertical and the other two horizontal. The value of the 3-form on such a triple is equal to the product of the value of ω on the vertical vector with the value of dω on the pair of horizontal vectors. The first factor is not zero, so the second must be zero, and thus the form dω is zero on a plane of the field. □
In fact, the set of contact elements with a fixed point of contact is the set of all (n − 1)-dimensional subspaces of an n-dimensional vector space, i.e., a projective space of dimension n − 1. To give a contact element we must therefore give the n coordinates of the point of contact together with the n − 1 coordinates defining a point of an (n − 1)-dimensional projective space—2n − 1 coordinates in all.
Proof. A contact element is given by a 1-form on the tangent space, for which this element is a zero level set. This form is not zero, and it is determined up to multiplication by a nonzero number. But a form on the tangent space is a vector of the cotangent space. Therefore, a nonzero form on the tangent space, determined up to a multiplication by a nonzero number, is a nonzero vector of the cotangent space, determined up to a multiplication by a nonzero number, i.e., a point of the projectivized cotangent space. □
Problem. Does this field of planes have integral manifolds of higher dimensions?Answer. No.Problem. Is it possible to give the field of contact hyperplanes by a differential 1-form on the manifold of all contact elements?Answer. No, even if the underlying n-dimensional manifold is a euclidean space (for example, the ordinary two-plane).
Proof of Theorem. Since the assertions of the theorem are local, it is sufficient to prove it in a small neighborhood of a point of the manifold. In a small neighborhood of a point on a contact manifold, a field of contact planes can be given by a differential form ω on the contact manifold. We fix such a 1-form ω.By the same token we can represent the symplectified space of the contact manifold over our neighborhood as the direct product of the neighborhood and the line minus a point. Namely, we associate to the pair (x, λ)—where x is a point of the contact manifold and λ is a nonzero number—the contact form given by the differential 1-form λω on the tangent space at the point x. Thus in the part of the symplectified space we are considering, we have defined a function λ whose values are nonzero numbers. It should be emphasized that λ is only a local coordinate on the symplectified manifold and that this coordinate is not defined canonically; it depends on the choice of differential 1-form ω. The canonical 1-form α can be written in our notation asand does not depend on the choice of ω. The exterior derivative of the 1-form α thus has the formWe will show that the 2-form dα is nondegenerate, i.e., that for any vector ξ tangent to the symplectification, we can find a vector η such that dx (ξ, η) ≠ 0. We select from vectors tangent to the symplectification, those of the following type. We call a vector ξ vertical if it is tangent to the fiber, i.e., if π✱ξ = 0. We call the vector ξ horizontal if it is tangent to a level surface of the function λ, i.e., if dλ(ξ) = 0. We call the vector ξ a contact vector if its projection onto the contact manifold lies in the contact plane, i.e., if ω(π✱ξ) = 0 (in other words, if α(ξ) = 0).We calculate the value of the form dα on a pair of vectors (ξ, η):Assume that ξ is not a contact vector. For η, take a nonzero vertical vector, so that π✱η = 0. Then the second term is equal to zero, and the first term is equal towhich is not zero since η is a nonzero vertical vector and ξ is not a contact vector. Thus if ξ is not a contact vector, we have found an η for which dα(ξ, η) ≠ 0.Now assume that ξ is a contact vector and not vertical. Then for η we take any contact vector. Now the first term is entirely zero, and the second (and therefore the sum) is reduced to λ dω(π✱ξ, π✱η). Since ξ is not vertical, the vector π✱ξ lying in the contact plane is not zero. But the 2-form dω is nondegenerate on the contact plane (by the definition of contact structure). Thus there is a contact vector η such that dω(π✱ξ, π✱η) ≠ 0. Since λ ≠ 0, we have found a vector η for which dα(ξ, η) ≠ 0.Finally, if the vector ξ is nonzero and vertical, then for η we can take any vector which is not a contact vector. □
Proof. Assume that the field is degenerate, i.e., that there exists a nonzero vector ξ′ in a plane of the field such that dω(ξ′, η′) = 0 for all vectors η′ in this plane. For such a ξ′, the quantity dω(ξ′, η′) as a function of η′ is a linear form, identically equal to zero on the plane of the field. Therefore there is a number μ not dependent on η′ such thatfor all vectors η′ of the tangent space.We now take for ξ a tangent vector to the symplectified manifold for which π✱ξ = ξ′. Such a vector ξ is determined up to addition of a vertical summand, and we will show that for a suitable choice of this summand we will haveThe first term of the formula for dα is equal to dλ(ξ)ω(π✱η) (since ω(π✱ξ) = 0). The second term is equal to λ dω(π✱ξ, π✱η) = λμω(π✱η). We choose the vertical component of the vector so that dλ(ξ) = −λµ. Then ξ will be skew-orthogonal to all vectors η.Thus if dα is a symplectic structure, then the underlying field of hyperplanes is a contact structure. □
Proof. The assertion of the theorem follows from the fact that the canonical 1-form, the symplectic 2-form, and the action of the group of real numbers are all determined by the contact structure itself (for their construction we did not use coordinates or any other noninvariant tools), and the diffeomorphism f preserves the contact structure. It follows from this that f! preserves all that which was invariantly constructed using the contact structure, in particular the 1-form α, its derivative dα, and the action of the group.
Proof. Every diffeomorphism which commutes with the action of the multiplicative group projects onto some diffeomorphism of the contact manifold. To show that this is a contact diffeomorphism it is sufficient to prove the second assertion of the theorem (since only those vectors for which α(ξ) = 0 project onto the contact plane).To prove the second assertion we express the integral of the form along any path γ in terms of the symplectic structure dα:where the 2-chain σ(ε) is obtained from γ by multiplication by all numbers in the interval [ε, 1]. The boundary of σ contains, besides γ, two vertical intervals and the path εγ. The integrals of α over the vertical intervals are equal to zero, and the integral over εγ approaches 0 as ε does.Now from the invariance of the 2-form dα and the commutativity of our diffeomorphism F with multiplication by numbers it follows that for any path γand thus the diffeomorphism F preserves the 1-form α. ☐
Proof of Darboux’s theorem. We symplectify our manifold. On this new (2n + 2)-dimensional symplectic manifold there are a canonical 1-form α, a nondegenerate 2-form dα, a projection π onto the underlying contact manifold and a vertical direction at every point.The given differential 1-form ω on the contact manifold defines a contact form at every point. These contact forms form a (2n + 1)-dimensional submanifold of the symplectic manifold. The projection π maps this submanifold diffeomorphically onto the underlying contact manifold, and the verticals intersect this submanifold at a nonzero angle.Consider a point in the surface just constructed (in the symplectic manifold) lying over the point of the contact manifold we are interested in. In the symplectic manifold we can choose a local system of coordinates near this point such thatand such that the coordinate surface p0 = 0 coincides with our (2n + 1)-dimensional manifold (cf. Section 43, where in the proof of the symplectic Darboux’s theorem the first coordinate may be chosen arbitrarily).We note now that the 1-form p0 dq0 + ⋯ pn dqn has derivative dα. Thus, locally,where w is a function which can be taken to be zero at the origin. In particular, on the surface p0 = 0 the form α takes the formThe projection π allows us to carry the coordinates p1,..., pn; q0; q1,..., qn and the function w onto the contact manifold. More precisely, we define functions x, y, and z by the formulaswhere A is a point on the surface p0 = 0.Then we obtainand it remains only to verify that the functions (x1,..., xn; y1,..., yn; z) form a coordinate system. For this it is sufficient to verify that the partial derivative of w with respect to q0 is not zero, or in other words that the 1-form α is not zero on a vector of the coordinate direction q0. The latter is equivalent to the 2-form dα being nonzero on the pair of vectors: the basic vector in the direction of q0 and the vertical vector.But a vector in the coordinate direction q0 is skew-orthogonal to all vectors of the coordinate plane p0 = 0. If it was also skew-orthogonal to the vertical vector, then it would be skew-orthogonal to all vectors, which contradicts the nondegeneracy of dα. Thus ∂w/∂q0 ≠ 0, and the theorem is proved. ☐
Proof. We use the expression for the increment of the ordinary hamiltonian function over a path in terms of the vector field and the symplectic structure (Section 48C). For this we draw a vertical interval {λB}, 0 < λ ≤ 1, through the point B of the symplectification at which we want to calculate the hamiltonian function. The translations of this interval over small time τ under the action of the symplectified flow defined by our field X, fill out a two-dimensional region σ(τ). The value of the hamiltonian at the point B is equal to the limitsince H(λB) → 0 as λ → 0. But the integral of the form dα over the region is the integral of the 1-form α along the edge formed by the trajectory of the point B (the other parts of the boundary give zero integrals). Therefore, the double integral is simply the integral of the 1-form α along the interval of trajectories, and the limit is the value of α on the velocity vector Y of the symplectified field. Thus K(πB) = H(B) = α(Y) = ω(X), as was to be shown. ☐
Solution. A point of the symplectification can be given by the 2n + 2 numbers xi, yi, z, and λ, where (x, y, z) are the coordinates of a point of the contact manifold and λ is the number by which we must multiply ω to obtain the given point of the symplectified space.In these coordinates α = λx dy + λ dz. Therefore, in the coordinate system p, q, wherethe form α takes the standard form:The action Tμ of the multiplicative group is now reduced to multiplication of p by a number:The contact hamiltonian K can be expressed in terms of the ordinary hamiltonian H = H(p, q, p0, q0) by the formulaThe function H is homogeneous of degree 1 in p. Therefore, the partial derivatives of K at the point (x, y, z) are related to the derivatives of H at the point (p = x, p0 = 1, q = y, q0 = z) by the relationsHamilton’s equations with hamiltonian function H therefore have the following form at the point under consideration:from which we obtain the answer above.
Solution. In the notation of the solution of the preceding problem we must express the ordinary Poisson bracket of the homogeneous hamiltonians H and H′ at the point (p = x, p0 = 1, q = y, z0 = z) in terms of the contact hamiltonians K and K′. We haveSubstituting the values of the derivatives from the preceding problem, we find at the point under consideration
Problem 1. Lay out an interval of length t on every interior normal to an ellipse in the plane. Draw the curve obtained and investigate its singularities and its transitions as t changes.Problem 2. Do the same thing for a triaxial ellipsoid in three-dimensional space.
Example. Let V be a smooth manifold and G some Lie group of diffeomorphisms of V. Since every diffeomorphism takes 1-forms on V to 1-forms, the group G acts on the cotangent bundle M = T*V.Recall that on the cotangent bundle there is always a canonical 1-form α (“pdq”) and a natural symplectic structure ω = dα. The action of the group G on M is symplectic since it preserves the 1-form α and hence also the 2-form dα.A one-parameter subgroup {gt} of G defines a phase flow on M. It is easy to verify that this phase flow has a single-valued hamiltonian function. In fact, the hamiltonian function is given by the formula from Noether’s theorem:
Remark. The appearance of the constant C in this formula is a consequence of an interesting phenomenon: the existence of a two-dimensional cohomology class of the Lie algebra of (globally) hamiltonian fields.The quantity C(a, b) is a bilinear skew-symmetric function on the Lie algebra. The Jacobi identity gives usA bilinear skew-symmetric function on a Lie algebra with this property is called a two-dimensional cocycle of the Lie algebra.If we choose the constants in the hamiltonian functions differently, then the cocycle C is replaced by C′, wherewhere ρ is a linear function on the Lie algebra. Such a cocycle C′ is said to be cohomologous to the cocycle C. A class of cocycles which are cohomologous to one another is called a cohomology class of the Lie algebra.Thus, a symplectic action of a group G for which single-valued hamiltonians exist defines a two-dimensional cohomology class of the Lie algebra of G. This cohomology class measures the deviation of the action from one in which the hamiltonian function of a commutator can be chosen equal to the Poisson bracket of the hamiltonian functions.
Example. Let V be a smooth manifold and G a Lie group acting on V as a group of diffeomorphisms. Let M = T*V be the cotangent bundle of the manifold V with the usual symplectic structure ω = dα. The hamiltonian functions of one-parameter groups are defined as above:(1)
Theorem. This action is Poisson.Proof. By definition of the 1-form α, the hamiltonian functions Ha are linear “in p” (i.e., on every cotangent space). Therefore, their Poisson brackets are also linear. Thus the function H[a, b] − (Ha, Hb) is linear in p. Since it is constant, it is equal to zero. ☐In the same way, we can show that the symplectification of any contact action is a Poisson action.Example. Let V be three-dimensional euclidean space and G the six-dimensional group of its motions. The following six one-parameter groups form a basis of the Lie algebra: the translations with velocity 1 along the coordinate axes q1, q2, and q3 and the rotations with angular velocity 1 around these axes. By formula (1), the corresponding hamiltonian functions are (in the usual notation) p1, p2, p3; M1, M2, M3, where M1 = q2 p3 − q3 p2, etc. The theorem implies that the pairwise Poisson brackets of these six functions are equal to the hamiltonian functions of the commutators of the corresponding one-parameter groups.
Example. Let V be a smooth manifold, G a Lie group acting on V as a group of diffeomorphisms, M = T*V the cotangent bundle and Ha the hamiltonian functions constructed above of the action of G on M (cf. (1)).Then the “momentum” mappingcan be described in the following way. Consider the map Φ: G → M given by the action of all the elements of G on a fixed point x in M (so Φ(g) = gx). The canonical 1-form α on M induces a 1-form Φ*α on G. Its restriction to the tangent space at the identity of G is a linear form on the Lie algebra.
Thus to every point x in M we have associated a linear form on the Lie algebra. It is easy to verify that this mapping is the momentum of our Poisson action.In particular, if V is euclidean three-space and G is the group of rotations around the point 0, then the values of the momentum are the usual vectors of angular momentum; if G is the group of rotations around an axis, then the values of the momentum are the angular momenta relative to this axis; if G is the group of parallel translations, then the values of the momentum are the vectors of linear momentum.
Remark. These conditions can be weakened. For example, instead of compactness of the group Gp we can require that the action be proper (i.e., that the inverse images of compact sets under the mapping (g, x) → (g(x), x)are compact). For example, the actions of a group on itself by left and right translation are always proper.
Proof. A vector ζ lies in the skew-orthogonal complement to the tangent plane of an orbit of the group G if and only if the skew-scalar product of the vector ζ with velocity vectors of the hamiltonian flow of the group G is equal to zero (by definition). But these skew-scalar products are equal to the derivatives of the corresponding hamiltonian functions in the direction ζ. Therefore, the vector ζ lies in the skew-orthogonal complement to the orbit of G if and only if the derivative of the momentum in the direction ζ is equal to zero, i.e., if ζ lies in T(Mp). ☐The representatives ξ′ and η′ are defined up to addition of a vector from the tangent plane to the orbit of the group Gp. But this tangent plane is the intersection of the tangent planes to the orbit Gx and to the manifold Mp (by the last theorem of part A). Consequently, the addition to ξ′ of a vector from T(Gpx) does not change the skew-scalar product with any vector η′ from T(Mp) (since by the lemma T(Gpx) is skew-orthogonal to T(Mp)). Thus, we have shown the independence from the representatives ξ′ and η′.The independence of the quantity [ξ, η]p from the choice of the point x of the orbit f follows from the symplectic nature of the action of the group G on M and the invariance of Mp. Thus we have defined a differential 2-form on Fp:It is nondegenerate, since if [ξ, η]p = 0 for every η, then the corresponding representative ξ′ is skew-orthogonal to all vectors in T(Mp). Therefore, ξ′ must be the skew-orthogonal complement to T(Mp) in T M. Then by the lemma ξ′ ∈ T(Gx), i.e., ξ = 0.The form Ωp is closed. In order to verify this we consider a chart, i.e., a piece of submanifold in Mp, transversally intersecting the orbit of the group Gp in one point.The form Ωf is represented in this chart by a 2-form induced from the 2-form ω which defines the symplectic structure in the whole space M, by means of the embedding of the submanifold piece. Since the form ω is closed, the induced form is also closed. The theorem is proved. ☐
Proof. Let π: V → W be the factorization map, and ω ∈ T*W a 1-form on W at the point w = πv. The form π*ω on V at the point v belongs to M0 and projects to a point in the quotient F0. Conversely, the elements of F0 are the invariant 1-forms on V which are equal to zero on the orbits; they define 1-forms in W. We have constructed a mapping T*W → F0; it is easy to see that this is a symplectic diffeomorphism.The case p ≠ 0 is reduced to the case p = 0 as follows. Consider a riemannian metric on V, invariant with respect to G. The intersection of Mp with the cotangent plane to V at the point v is a hyperplane. The quadratic form defined by the metric has a unique minimum point S(v) in this hyperplane. Subtraction of the vector S(v) carries the hyperplane Mp ∩ T*Vv into M0 ∩ T*Vv, and we obtain a possibly nonsymplectic diffeomorphism Fp → F0.The difference between the symplectic structures on T*W induced by that of Fp and F0 is a 2-form, induced by a 2-form on W. ☐
Proof. The relation defining a hamiltonian field XH with hamiltonian H on a manifold M with form ωimplies an analogous relation for the reduced field in view of the definition of the symplectic structure on Fp. ☐
The transition to the reduced phase space in this case is almost by “elimination of the cyclic coordinate φ.” The difference is that the usual procedure of elimination requires that the configuration or phase space be a direct product by the circle, whereas in our case we have only a bundle. This bundle can be made a direct product by decreasing the size of the configuration space (i.e., by introducing coordinates with singularities at the poles); the advantage of the approach above is that it makes it clear that there are no real singularities (except singularities of the coordinate system) near the poles.
Proof. It is clear that a phase curve which is an orbit projects to a point. If a phase curve x(t) projects to a point, then it can be expressed uniquely in the form x(t) = g(t)x(0), and it is then easy to see that {g(t)} is a subgroup. ☐
Proof. The critical points of the mapping P × H are the conditional extrema of H on the momentum level manifold Mp (since this level manifold is regular, i.e., for every x in Mp, we have).
After factorization by Gp, the conditional extrema of H on Mp define the critical points of the reduced hamiltonian function (since H is invariant under Gp). ☐
For example, for a system with one degree of freedom the normal form of degree 2m (or 2m + 1) looks likeand for a system with two degrees of freedom the Birkhoff normal form of degree 4 will beThe coefficients a1 and a2 are characteristic frequencies, and the coefficients aij describe the dependence of the frequencies on the amplitude.
The fact that, in a rotating coordinate system, the truncated system is autonomous is very good luck. The total system of Hamilton’s equations (including terms of degree higher than three in the hamiltonian) is not only not autonomous in a rotating coordinate system, but is not even 2π-periodic (but only 6π-periodic) in time. The autonomous system with hamiltonian H0 is essentially the result of averaging the original system over closed trajectories of the linear system with ε = 0 (where we disregard terms of degree higher than three).
The magnitude of the splitting of separatrices is exponentially small for small ε; therefore it is easy to overlook the phenomenon of splitting in calculations in one or another scheme of “perturbation theory.” However, this phenomenon is very important in fundamental questions. For example, its existence immediately implies the divergence of the series in numerous versions of perturbation theory (since if the series converged, there would be no splitting).In general, the divergence of series in perturbation theory (while a good approximation is given by a few initial terms) is usually related to the fact that we are looking for an object which does not exist. If we try to fit a phenomenon to a scheme which actually contradicts the essential features of the phenomenon, then it is not surprising that our series diverge.The Birkhoff series (which are obtained if one continues infinitely the normalizations of the initial terms of the Taylor series of the hamiltonian function) are one example of a formally convergent, but actually divergent, scheme of perturbation theory. If these series converged, then a general oscillating system with one degree of freedom with periodic coefficients would be reduced near an equilibrium position to an autonomous normal form and there would be no splitting of separatrices in it (whereas in fact there is).
The case of resonance of fourth order is somewhat exceptional. In this case, in the normal form there are both resonant and non-resonant terms of order 4. The shape of the phase curves of the truncated system depends on which of these terms of the normal form dominates, a resonant one or a non-resonant one. In the first case the development is the same as for third-order resonance, except that in place of a triangle there is a square. In the second case the development is the same as for n > 4.
Note that the question of the behavior of solutions of the equations of motion on an infinite time interval has only an indirect relation to the problem of the motion of real planets. The reason is that, after intervals of billions of years, small non-conservative effects not considered in Newton’s equations become important. Thus, the effects of the gravitational interaction of the planets are of real importance only when they seriously change the picture of motion within a finite time which is small in comparison with the time of development of non-conservative effects.In calculating motion over such finite times, computers prove to be very useful, quickly determining the motion of the planets for many thousands of years in the future or past. However, we should note that even the application of modern calculating methods may be insufficient to predict the influence of perturbations if a phase point falls in the zone of exponential instability.Asymptotic and qualitative methods have even greater value for the study of charged particles in magnetic fields, since in this situation a particle outstrips the computer and makes so many orbits that mechanical calculation of its trajectory is impossible even in the absence of exponential instability.
Thus, for example, in their motion around the sun, Jupiter and Saturn, in one day, go through approximately 299 and 120.5 seconds of arc respectively. Therefore, the denominator 2ωJ – 5ωs is very small in comparison with each of their frequencies. This amounts to a large long-period perturbation of the planets on one another (its period is about 800 years); the study by Laplace of this effect was one of the first successes of the theory of perturbations.
The conditions for nondegeneracy and isoenergetic nondegeneracy are independent from one another; i.e., a nondegenerate system could be isoenergetically degenerate, and an isoenergetically nondegenerate system could be degenerate. In the many-dimensional case (n > 2) isoenergetic nondegeneracy means nondegeneracy of the following mapping of the (n − 1)-dimensional level manifold of the function H0 of n action variables to the projective space of dimension n − 1:![]()
We note that the fact that the perturbations are hamiltonian is essential, since for non-conservative perturbations it is clear that the action variables may evolve. In celestial mechanics, their evolution means a secular change in the major semi-axes of the keplerian ellipses, i.e., the planets falling into the sun, colliding, or escaping to a large distance in a time which is inversely proportional to the size of the perturbation. If conservative perturbations led to evolutions in a first approximation, this would manifest itself in the fate of the planets after a time on the order of 1,000 years. Fortunately, the order of magnitude of the non-conservative perturbations is much less.
We note that the motion of the Laplace vector of the earth is, apparently, one of the factors involved in the occurrence of ice ages. The reason is that, when the eccentricity of the earth’s orbit increases, the time it spends near the sun decreases, while the time it spends far from the sun increases (by the law of areas); thus the climate becomes more severe as the eccentricity increases. The magnitude of this effect is such that, for example, the amount of solar energy received in a year at the latitude of Leningrad (60°N) may attain the value which now corresponds to the latitudes of Kiev (50°N) (for decreased eccentricity) and Taimir (80°N) (for increased eccentricity). The characteristic time of variation of the eccentricity (tens of thousands of years) agrees well with the interval between ice ages.
We note that the theorem about the annulus would follow from the theorem about the torus if in the latter we could throw out the condition on the eigenvalues. In fact, we can put together a torus from two copies of our annulus, inserting a narrow connecting annulus along each of the two boundary circles.Then we can extend our mapping of the annulus to a symplectic diffeomorphism of the torus such that: (1) on each of the two large annuli the diffeomorphism coincides with the original, (2) on each of the connecting annuli the diffeomorphism has no fixed points, and (3) the center of gravity remains fixed.The construction of such a diffeomorphism of the torus uses the property that the boundary circles rotate in different directions. On each connecting annulus all points are translated in the same direction as on both circles bounding the connecting annulus. Since the translations on the connecting annuli are in opposite directions, the size of the translations can be chosen to ensure preservation of the center of gravity.Now out of four fixed points on the torus, two must lie in the original annulus, and we obtain the theorem on annuli from the theorem on tori.
A restriction of this sort may be necessary in higher-dimensional problems. It is not impossible that Poincaré’s theorem is due to an essentially two-dimensional effect, as is the following theorem of A. I. Shnirel’man and N. A. Nikishin: every area-preserving diffeomorphism of the two-dimensional sphere to itself has at least two geometrically different fixed points.The proof of this theorem is based on the fact that the index of the gradient vector field of a smooth function of two variables at an isolated critical point cannot be greater than 1 (although it can be equal to 1, 0, −1, −2, −3,...), and the sum of the indices of all the fixed points of an orientation-preserving diffeomorphism of the two-dimensional sphere to itself is equal to 2. On the other hand, the index of the gradient of a smooth function of a large number of variables at a critical point can take any integer value.
The proof could be reduced to investigating the intersection of two lagrangian submanifolds of a 4n-dimensional space (ℝn × Tn × ℝn × Tn) with Ω = dx ⋀ dy − dX ⋀ dY, one of which is the diagonal (X = x, Y = y) and the other the graph of the mapping AN.However, it is easier to directly construct a suitable function on the torus. In fact, the mappinghas the form
By the implicit function theorem, the mapping AN has, near the torus x = x0, a torus which is displaced only radially ((x, y) → (X, Y)) and is given by an equation of the form x = f(y); its image is also given by an equation x = g(y) of the same form. In this notation, X(f(y), y) = g(y), Y(f(y), y) = y.Since A is homologous to the identity, it follows that AN has a single-valued global generating function of the form Xy + S(X, y), where S has period 2π in the variable y.The function F(y) = S(X(f(y), y), y) has at least 2n critical points yk on the torus. All the points ξk = ( f (yk), yk ) are fixed points for AN. In fact,Therefore, since dF |yk = 0, it follows that![]()
, as was to be shown.
This “paradox” becomes, perhaps, clearer from the following calculation. The quadratic forms Ax2 + 2Bxy + Cx2 with different eigenvalues form a submanifold of the three-dimensional space with coordinates A, B, and C, given by one equation λ1 − λ2 = 0, where λ1 . 2(A, B, C) are the eigenvalues. However, the left-hand side of this equation is the sum of two squares, as is clear from the formula for the discriminant of the characteristic equation:Thus the single equation Δ = 0 determines a line in the three-dimensional space of quadratic forms (A = C, B = 0), and not a surface.
Consider, for example, a convex surface in three-dimensional euclidean space. The second fundamental form of the surface determines an ellipse in the tangent space at every point. Therefore, we have a two-parameter family of ellipses (which can be translated to one plane by choosing a local coordinate system near a point on the surface). We come to the conclusion that, at every point of the surface except at certain isolated points, the ellipse has axes of different lengths. Therefore, on surfaces of general form, there are two orthogonal fields of directions (the major and minor axes of the ellipses) with isolated singular points. In differential geometry these directions are called the directions of principal curvature, and these singular points are called umbilical points. For example, on the surface of an ellipsoid there are four umbilical points; they lie on the ellipse containing the major and minor axes, and two of them are clearly visible in the picture of the geodesics on an ellipsoid (cf. Figure 207).
Problem. Classify the characteristic oscillations of a system with the symmetries of an equilateral triangle (allowing not only rotation by 120°, but also reflection through the altitude of the triangle).Problem. Classify the characteristic oscillations of a system whose group of symmetries is the group of 24 rotations of the cube.Answer . The oscillations will be of five types. By rotations, from each oscillation one can obtain systems of 8, or 6, or 4, or 2, or 1 independent oscillations (in the last case the oscillations are entirely symmetric).Remark. To classify oscillations in systems with any group of symmetries, a special apparatus has been developed (the so-called theory of group representations). Cf., for example, Michael Tinkham, Group Theory and Quantum Mechanics, McGraw-Hill, 1964.
The exact formulation of this assumption is that, for all membranes except a set of infinite codimension, the mapping from the space of symmetric membranes into the space of symmetric ellipsoids is transverse to each of the manifolds of ellipsoids with a given number of multiple axes.
The problem of connecting the geometry of a membrane with the properties of its characteristic oscillations has been intensively studied in recent years by many authors (including H. Weyl, S. Minakshisundaram and A. Pleijel, A. Selberg, J. Milnor, M. Kac, I. Singer, H. McKean, M. Berger, Y. Colin de Verdière, J. Chazarain, J. J. Duistermaat, V. F. Lazutkin, A. I. Shnirel’man, and S. A. Molchanov).To the simplest question, “Can you hear the shape of a drum?” the answer turns out to be negative: there exist non-isometric riemannian manifolds with the same spectrum. On the other hand, several properties of a manifold can be recovered from the eigenvalues of the laplacian and from the properties of eigenfunctions (for example, the complete set of lengths of closed geodesics can be recovered).
J. Heading, Introduction to phase integral methods, Methuen Co. Ltd., 1962. (Cf. especially Appendix II (by V. P. Maslov) in the Russian translation of Heading’s book, Moscow 1965).
V. P. Maslov, Théorie des perturbations et méthodes asymptotiques, Pairs, Dunod, 1972 (Russian edition: Moscow University, 1965).
V. I. Arnold, On a characteristic class entering into conditions of quantization, Functional Analysis and its Applications, v. I (1967).
L. Hörmander, Fourier integral operators, Acta Math. 127 (1971), 79–183.
The nondegeneracy condition is satisfied for almost all points Q. Those exceptional points Q for which it is not satisfied form a set of measure zero in the configuration space. In the general case, this set is a surface whose dimension is one less than the dimension of the configuration space. This surface, playing the role of a caustic in our problem, can itself have complicated singularities.
We note that the definitions of focal points to M and the Morse index do not depend on Schrödinger’s equation, but relate simply to the geometry of the phase flow in the cotangent bundle to the configuration space (or to the calculus of variations, which is the same thing).In particular, as our lagrangian manifold M we may take the fiber of the cotangent bundle passing through the point (p0, q0) (given by the condition q = q0). In this case a focal point to M on the phase curve going out from (p0, q0) is called conjugate to the original point (more precisely, the projection of this focal point onto the configuration space is said to be conjugate to the point q0 along the extremal in the configuration space starting at q0 with momentum p0). In the even more special case of motion along a geodesic on a riemannian manifold, a focal point to a fiber of the cotangent bundle is called conjugate to the initial point of the geodesic along this geodesic. For example, the south pole of a sphere is conjugate to the north pole along any meridian.The Morse index of an interval of a geodesic, equal to the number of points conjugate to the initial point, plays an important role in the calculus of variations. Namely, we consider the second differential of the action as a quadratic form on the space of variations (with fixed end-points) of the geodesic we are studying. Then the index of inertia of this quadratic form is equal to the Morse index (cf., for instance, J. Milnor, Morse Theory, Princeton University Press, 1967).Thus the geodesic, up to the first conjugate point, is a minimum of the action, which justifies the name “principle of least action” for various variational principles of mechanics.
We should point out that, among the pieces of various ranks into which the set of singular points is divided, there is no piece of dimension n − 2. After the simplest singular points, forming a manifold of dimension n − 1, there are the points where the rank drops by two; they form a manifold of dimension n − 3. The projection of the set of singular points onto the configuration space (the caustic) consists, in general, of pieces of all dimensions from 0 to n − 1 without omissions.
We note that it is necessary to prove that the definitions of positive direction near different points agree with one another. Furthermore, it must be shown that the positive direction near one point is well defined, i.e., does not depend on the coordinate system. All this can be done by direct calculations (cf. the article cited above in “Functional Analysis”). For further development of these ideas, see V. I. Arnold, Sturm theorems and symplectic geometry, Funct. Anal. Appl. 19 (1985).
Problem. Find the index of the circle p = cos t, q = sin t oriented by the parameter t, 0 ≤ t ≤ 2π, in the lagrangian manifold p2 + q2 = 1 of the phase plane.
Answer. + 2.
The eigenfunctions corresponding to these eigenvalues are also associated with lagrangian manifolds, but this association is not so simple. In fact, we cannot write down asymptotic formulas for eigenfunctions, but only for functions approximately satisfying the equations of characteristic functions. These functions turn out to be small outside the projection of the lagrangian manifold onto the configuration space. The asymptotic formulas have singularities near the caustics formed by the projection.The actual eigenfunctions, however, can behave entirely differently, at least if the eigenvalue is multiple or if there are eigenvalues close to it (cf. Appendix 10).
V. I. Arnold, Singularities of smooth mappings, Russian Math. Surveys 23: 1 (1968) 1–44.
Symposium on Singularities of Smooth Manifolds and Maps, Univ. of Liverpool, 1969–70.Proceedings. Springer, 1971. See especially the article of R. Thom and H. Levine.
Golubitsky and Guillemin, Stable Mappings and Their Singularities, Springer-Verlag, 1973.
V. I. Arnold, Normal forms for functions near degenerate critical points, the Weyl groups of Ak, Dk, Ek, and lagrangian singularities, Functional Analysis and Its Applications 6:4 (1972) 254–272.V. I. Arnold, Critical points of smooth functions and their normal forms, Uspekhi Mat. Nauk 30:5 (1975).
We note that this assertion about two-dimensional lagrangian mappings does not follow from the classification theorem for general (non-lagrangian) mappings. In the first place, lagrangian mappings make up a very restricted class among all smooth mappings, and therefore they can (and actually do for n > 2) have as typical, singularities which are not typical for mappings of general form. Secondly, the possibility of reducing a mapping to normal form by diffeomorphisms of the pre-image and image does not imply that this can be done using a lagrangian equivalence.
Other applications of the formulas of this section can be found in the theory of Legendre singularities, i.e., singularities of wave fronts. Legendre transforms, envelopes, and convex hulls (cf. Appendix 4). The theories of lagrangian and Legendre singularities have direct application, not only in geometric optics and the theory of asymptotics of oscillating integrals, but also in the calculus of variations, in the theory of discontinuous solutions of nonlinear partial differential equations, in optimization problems, pursuit problems, etc. R. Thom has suggested the general name catastrophe theory for the theory of singularities, the theory of bifurcations, and their applications.
To obtain a soliton moving with velocity c, it is sufficient to substitute the function u = φ(x − ct) into equation (1). Then we obtain the equation φ″ = 3φ2 + cφ + d for φ (d is a parameter). This is Newton’s equation with a cubic potential. There is a saddle on the phase space (φ, φ′). The separatrix going from this saddle to the saddle for which φ = 0 determines a solution φ tending to 0 as x → ± ∞; it is a soliton.
The explicit form of the polynomials Ps and Qs, and also the explicit form of the action-angle variables (and therefore of solutions of equation (1)), is described in terms of solutions of the direct and inverse problems of scattering theory with potential u.The explicit form of the polynomials Qs can also be obtained from the following theorem of Gardner, generalizing Lax’s theorem. In the space of functions of x, we consider a differential operator of the form A = ∑ pi∂m − i, where p0 = 1, and the remaining coefficients pi are polynomials in u and the derivatives of u with respect to x. It turns out that, for any s there is an operator As of order 2s + 1 such that its commutator with the Sturm-Liouville operator L is the operator of multiplication by a function [L, As] = Qs.The operator As is defined by these conditions uniquely up to the addition of linear combinations of the Ar with r < s; in the same way, the polynomials Qs are determined up to the addition of linear combinations of the preceding Qr’s.